You are on page 1of 238
Reference Guide for the Foreign Pharmacy Licensing Exam er Perr health care delivery aok- Dik kako) spensing pharmacy administration health care economics Ue taste naan reign pharmacy by Manan H. Shroff, R.Ph Revised Edition Ssemerr seeeseeseeeseeseeeseesteeseeseeeseeseeeseeseeeeeereeeeeeeeeeeeeeeeeeeeeeeeerereeeeeeeeeeeeeeeeeeREEEEEEEEEEEE EE EEE Reference Guide for Foreign Pharmacy Licensing Exam Krisman Questions and Answers- 2nd Revised Edition TABLE OF CONTENTS SECTION-I QUESTIONS 7 SECTION-IT ANSWERS 121 SECTION-II TABLES 243 \www.pharmacyexam.com 6 Reference Guide for Foreign Pharmacy Licensing Exam Krisman Questions and Answers- 2nd Revised Edition 1 Benzyl alcohol is classified as: . Drug degradation d. Drug oxidation a. Emulsifying agent b. Preservative 7. Polymorphism is generally defined as ¢. Diluent a: d. Suspending agent a, Substance that may exist in more than 2. Cold cream is an example of: one crystalline form, a Suspension b. Substance that may exist only in meta- b. O/W emulsion stable form. c. W/O emulsion 4 OW/O emulsion ¢. Substance that has different viscosity time to time. 3. Egg yolk or egg white is used as: 4. Substance that reduces interfacial ten- a. Emulsifying agent sion. b. Suspending agent ©, Binder 8. The minimum concentration of a drug d. Preservative at the receptor site to initiate pharmacologi- cal action is defined as: 4. The transfer of a drug from high con- centrated areas to low concentrated areas is a. max generally defined as: b. MEC c. MTC a Infusion d ce b. Levigation ©, Diffusion 9. The area under curve gives useful in- 4. Dissolution formation about : 5. Which of the following is the most a. ‘The amount of drug systematically ab- suitable route for administration of insulin ? sorbed. a. IM b. The time to reach peak concentration. b Sc . ¢. The time to reach minimum toxic a IV bolus concentration. 6. Noyes Whitney equation is helpful to a. The concentration at which pharma- predict the rate of: cological actions of drug would be initiated. a. Drug diffusion b. _ Drugdissolution 10. Which of the following is the major plasma protein involved in drug binding? www.pharmacyexam.com a Reference Guide for Foreign Pharmacy Licensing Exam Questions and Answers- 2nd Revised Edition a. Globulin b. Creatinine c 4. Albumin Glycoprotein 11. Which of the following equations may be useful to find out the plasma concentration of adrug ? a -V,xP=C, b- PxC,=V, c V,= PI, da V,=CyP 12. The initial dose of a drug through IV bolus to achieve desirable plasma concentra- tion at once is known as: Loading dose Maintenance dose Replacement dose Degradation dose 13. Which of the following is/are useful to measure glomerular filtration rate? 1 Creatinine i Inulin I, Albumin a Tonly b. Land Tonly cc. Tand IIT only a All 14. The rapid degradation of 2 drug by liver enzymes in a liver is defined as: Third pass effect of metabolism First pass effect of metabolism Rapid degradation Liver elimination eege www.pharmacyexam.com 15. Krisman The normal renal creatinine clearance value lies between: aege 16. 200 to 300 ml/min 80 to 120 ml/min 30 to 60 ml/min 10 to 20 ml/min Which of the following is an example of an oligosaccharide? eaege Glucose Sucrose Starch Glycogen 17. Which pyrimidine base is found only in RNA? aege 18. aege 19, Cytosine ‘Thymine Uracif Adenine Heparin is classified as a(n): Heteropolysacharide Oligosaccharide Homopolysacharide Monosaccharide Ribonucleic acid exists in all of the following forms EXCEPT : 20. 1 RNA mRNA. qRNA URNA Which of the following structures is a host for Kreb’s cycle ? a b. Mitochondria Golgi bodies Reference Guide for Foreign Pharmacy Licensing Exam ‘Questions and Answers- 2nd Revised Edition ¢. Cytoplasmic membrane d. Ribosome 21. The synthesis of glucose fromsources other than carbohydrates is generally known a. Glycolysis b. —_ Gluconeogenesis c. — Glycogenolysis d.— Glucogenesis 22. Which of the following amino acids should be considered an essential amino acids) for the body ? 1 Phenylalanine TL. Leucine MI, Tryptophan a. Tonly b. Land Ionly cand Ilfonly a Ail 23. Which of the following enzymes ca- talyses the coupling of two molecules of nucleotides to form DNA ? a. Transferase b. Ligase c. Isomerase d. Aldehyde dehydrogenase 24. A nucleotide is a building block of: a. Sphingomide b. Nucleic acid c. Amino acid 4. Starch 25. — Which of the following cells are in- volved with immune responses of the body? www.pharmacyexam.com Krisman 1. B lymphocytes I. — T lymphocytes Ill. Neutrophils a. Tonly b. Land Ionly ¢. Hand Ill only da All 26. — Which of the following immunoglo- bulin levels are elevated during asthma ? a IgM b. IgD c. IgE 4. IgA 27 Allof the following tests are required to check sensitivity of class A weighing pre- scription balance EXCEPT : a. Armratio test b. Rider graduated beam test c. Shift test da Utest 28. The ratio of the mass of an object measured in a vacuum at specific tempera- ture to volume (in ml) of an object at the same temperature is defined as: Absolute density Specific gravity Relative density Apparent density eege 29, — The mean blood pressure of Mr. Ham is: 01/01/00 80 mmhg 04/04/00 90 mm hg. 01/02/00 82 mmhg 01/05/00 85 mmhg 01/03/00 81.5 mm hg 01/06/00 83 mm hg Reference Guide for Foreign Pharmacy Licensing Exam Questions and Answers- 2nd Revised Edition a BLS b. 85.6 c. 83.58 a 84.26 30. The deviation of data from its mean is generally described by: a, Theaverage b. The standard deviation c. The precision d ‘The accuracy 31. ‘The reproducibility of results of a number of experiments is generally known as a. Precision b. Bias c. Accuracy 4. — Closelessness 32. Ifthe value of p = 0.6 in binomial dis- tribution, what is the probability of failure ? a 02) be 04 Cn) a 10 33. The aterror is generally considered significant at: a, 1% bd 3% cof da 10% 34. — When the hypothetical value of a pa- the same as the observed value of a parameter, the error should be considered: a. Alfa-error b. —_Beta-error c. Gema-error a. Infinitive ‘www.pharmacyexam.com 35. Find out the degrees of freedom in a Chi-square testin a 2x2 contingency table (as- sume te: go sese segs 37. s are independent). RORe The F distribution generatly compares: ‘Two means ‘Two variances ‘Three means Three variances Which of the following elements has the highest electronegativity? eege 38. Which of the following molecules has the largest dipole movement? cH3 ca c. c CHB a FIGURE - I 10 Reference Guide for Foreign Pharmacy Licensing Exam Questions and Answers- 2nd Revised Edition CH3 H Nev a H CHB FIGURE-2 FIGURE-3 CHS ~ 4 ic F a c \ F FIGURE-4 a. Fig I b Fig c Fig iil ad Fig IV 39. Which of the following molecules has the highest boiling point? a HO b. .S c Hse 4 HCN 40. The bond between NH, and CO, is best described as a: www.pharmacyexam.com Krisman H oO a yo eege 41. Hydrophobic interaction force Ton dipole or jon induced dipole force London force Van der walls force The process of transforming a solid directly to a vapor state is generally defined Evaporation Melting Sublimation Levigation ‘The characteristic of solid substances toexhibit more than one crystalline or amor- phous form is defined as: aege 4B. Isomerism Polymorphism Zwitter ion Coupling Which of the following molecules rep- resents CIS form ? \ . a 7 ~ H cl FIGURE - I nu Reference Guide for Foreign Pharmacy Licensing Exam Krisman Questions and Answers- 2nd Revised Edition H c. Diffusion coefficient. 4. The length of the stagnant layer. a 46. Acetone is classified as a a. Polar solvent cl b. _ Nonpolar solvent c. Semipolar solvent 4. Dipolar solvent a 47 The process of degradation of ionic compounds into cations and anions in a pres- ence of water is defined as: va a. Solvation b. Hydration c Activation 4 Degradation 48. — What happens to the solubility of al- ee cohol as the molecular weight of alcohol in- i creases a FigI b. Fig II c Fig II a. Reduces ad. FigIV b. Increases c. Remain unchanged 4. Insoluble in wate 44, Which of the following drugs is an Insoluble in water angiotensin receptor antagonist ? Soe ae . light is classified as: a. Lisinopril b. Losartan c. Methyldopa & Oxidation 4. Captoprit b. Reduction cc. Photochemical degradation d. Racemization 45. — According to Fick’s law of diffusion, Which of the following is inversely propor- 54. The degradation of Penicillin G tionate to the rate of diffusion ? caine is highest in: a. The area of the solid. a. Solution b. The difference between the concentra- _ ae sion tion of solute to concentration of z a Tablet solute in stagnant layer. i www.pharmacyexam.com n Reference Guide for Foreign Pharmacy Licensing Exam Questions and Answers- 2nd Revised Edition 51 Therate of oxidation is influenced by all of the following EXCEPT: a. Temperature b. Radiation c. Presence of catalyst 4. Hydrolysis 52. — Which of the following are character- istics of pseudoplastic flow? 1 Viscosity of the flow generally de- creases with an inerease in the rate of shears. Il —_Noyyield value has been found with flow. HL Suspension of tragacanth follows the pseudoplastic’s flow. a Tonly b. Land Il only c. Mand If only 4. 3, S.and iit only 53. Which of the following is NOT true about microemulsion? a. Themean diameter of a droplet gener- ally lies between 1010 200 nm. b Itisathermodynamically stable system. ¢ —— Itrequires a cosurfactant. 4. Itis intermediate in property between solution and emulsion. 54, — Asystem with considerable interaction between dispersed phase and dispersion me- dium is known as: eege Radioactive colloids www.pharmacyexam.com Krisman 55, Tobalance the following equation, how ‘many molecules of NH,Cl would be required? (NH)),S+NICI,—> NIS + NH,CL aes hens 56. Therandom motion of solute particles in colloidal dispersion is known as: Newtonian flow Brownian motion Stoke’s law Non-Newtonian flow pose 57. Which of the following about floccu- lated suspension is NOT true? a, Particles of suspension form loose ag- gregates. b. Rate of sedimentation is very low. ¢. The time to form sediment is less. d. The sedimentation is easy to redis- perse. 58. The rate of sedimentation is indepen- dent of : a. The viscosity of dispersion medium. b. The diameter of suspended particles. ¢. The difference in densities between dispersed medium and dispersed phase. The lipophilic nature of particles. 59. — Which of the following compounds is an acetanilide? CH,CONHC,H, CH,CHO. C,H,CHEN.CH, C.H.N=NCH, eege 13, Reference Guide for Foreign Pharmacy Licensing Exam ‘Questions and Answers- 2nd Revised Edition 60. |The spontaneous isomerization of two stereoisomers in aqueous solution that causes specific rotation is known as: a, Zwitter ion rotation b. Micelle rotation c, Mutarotation 4, Steriorotation 61. Which of the following is a polysae- charide? a. Dextrose b. Dextran c. Lactose d. Sucrose 62. The susbstance that is isolated from the brain and produces fatty acid, galactose and sphingosine upon hydrolysis is known as: a. Sterols b. Phospholipids c.— Glycolipids d—— Saponins 63. Which of the following is NOT a by- drolyzed product of Jecithins? a Fatty acid b. Glycerol c. Phosphoric acid d. — Spingosine 64. N H CH.NHCOOH NH www.pharmacyexam.com Krisman Which of the following is anactive moiety of the above compound ? a. Carboxylic acid b. Imidazole e. Pyroline 4 Aniline 65. Albumin is an example of a: Simple protein Conjugated protein Derived protein Hydrolysed protein NH a | Ty Fluorination ( NH ° Fluorination of above compound will result into a well known cancer drug known as: a. Methadone b, — S-fluorouracil ¢. 6-mereaptopurine 4. Procainamide 67. For microbial assay of vitamin B-12, the test organism should be : a, Leichmani b.— Lplantarum c. aeruginosa 4. S.pneumonia 68. — Which of the following isNOT classified asa titrimetric method of analysis? a, Directtitration b. Gravimetric titration ©. Complexation reaction d.— Redox reaction 14 Reference Guide for Foreign Pharmacy Licensing Exam. ‘Questions and Answers- 2nd Revised Editi 69. Hemolytic anemia with abnormai he- moglobin is generally found in patients with : Vitarnin B-12 deficiency anemia Sickle cell anemia Iron deficiency anemia Folic acid deficiency anemia aege 70. Allof the following drugs may cause hemolysis in a patient with G6PD deficiency EXCEPT: Chloroquine Sulfonai Dimercaptrol Penicillin aoge 71. Which of the following drugs is use- ful in a Rh negative mother with a Rh posi- tive infant ? a, Rhogam b. — Octeroide acetate ¢. Immunoglobulin 4. Pheu-immune 72. — Which of the following is NOT true about PKU ? a. Ttisadisease usually characterized by ‘mental abnormalities. b. high concentration of phenylpyruvic- acid is found in urine, c. _Itoccurs due to excessive secretion of Phenylalanine hydroxylase enzyme. 4. A guthrie test is normally performed todetectiit. 73. The metabolite product ofepinephrine and norepinephrine is : a. Gama butyric acid b. —_Vanillymandelic acid cc. Homovanillic acid d. 5Hydroxyindoleacetic acid www-pharmacyexam.com Krisman 74, — Which of the following is ananaerobie organism? a, L. pneumonphllia b. Cl. tetani ©. N. meningitis d. E. coli 75. Which of the following organisms is responsible for causing most of the UTI? aS. pharyngitis 6. E.coli ¢. _N.gonorthea 4. T. palladium 76. The accurate diagnostic test for a pa- tient with cystic fibrosis is : a. Mantoux test b. Sweat test c. Breath test a. Sick test 77. ‘The allergic skin reaction character- ized by wheel formation is known as: a. Eczema b. Urticaria c. Impetigo a Exythema a. Itisadisease usually characterized by polyuria, polydypsia, and severe thirst. b. The urine volume sometimes increases 16 to 24 liters a day. c. This thought to occur due to over- activity of ADH. 4. A patient should be monitored for dehydration. 15 Reference Guide for Foreign Pharmacy Questions and Answers- 2nd Revised Edition 79. inwhich kinetic reaction is the rate of re- action independent from concentration ? a. First order b. Zero order c. Pseudo first order 4. Second order 80. The initial degradation of a drug by liver enzymes after oral administration of the drug is known as: Enzymatic degradation First pass metabolism Relative bioavailability Fick’s degradation aoe 81. Which of the following factors DOES. NOT affect the protein binding of a drug ? a. Theavailabitity of protein for binding. b. Binding affinity of protein to the drug. c. The presence of competing substances for protein binding. 4. The concentration of a drug at its receptor site. 82. In which of the following conditions is an increase in plasma protein albumin found 2 a, Severe burns b. Cystic fibrosis c. Trauma 4. Hypothyroidism 83. — Which of the following drugs is an H, re- ceptor antagonist ? a. Hydroxyzine b. Cimetidine . Diphenhydramine 4. Omeprazole ‘www.pharmacyexam.com nsing Exam 84, Whichof the following dugsis indicated forreducing elevated blood concentration of ammonia in blood ? a. Lactulose b. _Diphenoxylate c. Sueralfate 4 Calcium polycarbophill 85. Patients with hemophilia have a defi- ciency of a. Rhol) b. AHF c. ADH da ACE 86. Sodium polystyrene sulfonate is found to lower: Serum K* concentration Serum Na‘ concentration Serum Al" concentration ‘Serum Ca‘? concentration aoge 87. Which of the following cells are gen- erally found to be elevated in a patient with polycythemia vera? Reticulocytes Erythrocytes Leukocytes Thrombocytes Bose 88. Which of the following antihypertensive drugs acts by blockingalfa-I receptors? L Doxazosin ‘Terazosin I Prazosin Tonly Land Il only Hand Ill only 1,11, and If only eege 16 Reference Guide for Foreign Pharmacy Licensing Exam Questions and Answers- 2nd Revised Edition 89. Which of the following receptor's stimu- lationprevents the release of noradrenaline? a Alpha-I receptors >» Beta-) receptors cc. Alpha-2 receptors d. Beta 2 receptors 90. — Hypertrichosis is generally associated with the use of a. Hydralazine b. Minoxidil c. Methyldopa 4. Clonidine 91. The preferable route for Sodium Nitroprusside is a. Intramuscular b. Oral cc. Intravenous 4. Subcutaneous 92. An overdose of sodium nitroprusside generally causes a, Severe hypotension b. Hypertension c. Renal failure d. Severe edema 93. The use of Sodium Nitroprusside should be strictly restricted by a Adult men. = Adult women c. Neonates d. ‘Children 94, Whichofthe following hypertensive drugs is known as aninodilator ? www.pharmacyexam.com Krisman a. Nitroglycerin b. — Milrinone c. 4 Dipyridamole Digoxin 95, — Which of the following is a Class-1A arrhythmic agent ? a. Lidocaine b. —_Procainamide c. Encainide 4. Atenolol 96. Which of the following blood cho- lesterol lowering drugs is an HMG-COA in- hibitor ? Gembifrozil eege 97. A patient with acute hypercapnia should be treated with which of the following Doxapram Dopamine Disopyramide Ipecac 98. Which of the following drugs is found to be mucolytic or reduces the viscosity of mucous ? a. Dextromethorphan b. — Acetylcysteine cc. Terbutaline 4. Benzonatate 99. Which of the following drugsis indicated as uterine relaxant for women in labor ? Ephedrine b. ‘Terbutaline 17 Reference Guide for Foreign Pharmacy Licensing Exam Questions and Answers- 2nd Revised Edition cc. Isoetharine d.— Metaproterenol 100, Which of the following is an atropine - like drug? Retrovir Ipratropium Carvedilol Latanoprost aege 101, Which of the following is a centrally acting muscle relaxant ? a. Dantrolene b. Cyclobenzaprine c. Bromocriptine 4. Amphetamine 102. Which of the following anti- Parkinson’s drugs is a dopamine receptor ago- nist? a. Carbidopa b. — Benztropine ¢. Bromocriptine 4. Amantadine 103. Which of the following diuretics acts through inhibition of carbonic anhydrase en- zyme? a. Furosemide b. — Acetazolamide Spironolactone 4. Hydrochlorothiazide 104, Which ofthe following isa common ad- verse effect of metolazone? a Seizwe b. Electrolyte loss c S.LE. @—_Neuroleptic malignant syndrome www.pharmacyexam.com Krisman_ 105. Which of the following diuretics cause hyperkalemia when used concurrently with Captopril ? L— Amiloride IL Spironolactone I. Triamterene Tonly Land Il only Tl and Il only 1, U1, and IIT only eege 106. Probenecid may competitivety inhibit the renal tubular secretion of : - Methicillin Methotrexate UL Dapsone Tonty Land I only Mand I only J, U, and Ut only nore 107. Which of the following can be admin- istered for treatment of insulin overdose ? 1. Glucagon UW. LV. Dextrose UL. Lidocaine a Tonly b. and Ionly ©. and IlTonly 4. L,MandIiTonly 108. Which of the following sulfonylurea ‘agents is indicated for the treatment of diabe- ‘es insipidus? a, Glyburide b, —Chiiorpropamide ©. Glipizide 4. Tolbutamide 18 Reference Guide for Foreign Pharmacy Licensing Exam Krisman Questions and Answers- 2nd Revised Edition 109. The deficiency of vitamin A may cause a Osteoporosis b. —Nightblindness cc. Scurvy 4. Anemia 110. Which of the following can be used for the treatment of Methotrexate overdose ? a. Mephyton b.— Leucovorin Ca*® c. Pyridoxine a Niacin J11. Which of the following benzodiaz- epines can be safely administered to a geriat- ric patient ? a. Chlordiazepoxide b. Alprazolam .— Oxazepam 4. Prazepam 112. The active metabolite of Primidone is L Phenobarbital I. PEMA I. Trimethadione a Tonly b. Land Ionly c. and Ill only 4 I, I,and IT only 113. Which of the following drugs should not be used with Fluoxetine? a, Tranylcypromine b. Digoxin Amitriptyline d. Lidocaine www.pharmacyexam.com 114. Which of the following is not classified as an insect control chemical ? a. Insecticides b. Repellents ¢. Attractants 4. Antiseptics 115. Which of the following is NOT true about Barium sulfate? a. Itis medicinally used in roentgeno- graphy for the examination of the stomach and colon, b. _ Itisa clear solution, cc. Theprinciple adverse effectis constipation. 4d. _Ttneeds tobe mixed well with food or strained through gauze before it is administered to a patient. 116. The addition of Ascorbyl palmitate in the manufacturing process serves as a a. Preservative b. Antioxidant ©. Coloring agent 4. Flavoring agent 117. Epinephrine hydrochloride solution can be stabilized by adding a small amount of a. Sodium metabisulfite b. Sodium bisulfite c. Sulfur dioxide d. Potassium benzoate 118. Which of the following are classified as certified colors? I FDandC I. = Dand MM. External DandC 19 Reference Guide for Foreign Pharmacy Licensing Exam Questions and Answers- 2nd Revised Edition a Tonly b. Land only ce. Mand Mlfonly dL, Tand Ill only 119. Which of the following isnot considered aprimary taste? a Saline b. Sweet ©. Spicy da. Bitter 120. A 500.mg dose of a drug administered via LV. injection produces a plasma concentra- tion of 2.5 meg/ml after 16 hours. Ifthe initial plasma concentration of the drug is 10 meg/ml, bioavailability is 1,and volume of distribution is 120,000, what is the half-life of the drug? a 2hours: b. Bhours ce. Shours d.15hours 121. Sweet taste of acompoundis generally attributed to: Presence of Htions Presence of OH ions Presence of cations and anions Presence of alkaloids ees 122. The alcohol content of low iso-alco- hol elixir is : 8t0 10% 15 to 23 % 50 to 80 % 73 to 78% pe oe 123. Erythema muttiform is generally de- scribed as: www.pharmacyexam.com Krisman a, Thepresence oferythematous macules and papules. b. The presence of hair on skin ©. ‘The presence of large flaccid bullae on. skin, 4. The presence of scaling and sloughing on entire skin. 124. The antidote for Acetaminophen tox- icity is: a EDTA b. — Neacetyleysteine cc. Mesna 4. Diazepam 125. Which of the following drugs may cause cholestatic jaundice? [Chlorpromazine Il. Erythromycin estolate TI. Indomethacin Tonly Land II only Mand III only 1, IL, and III only pege 126. The principal adverse effect of Clinda- mycin is: a ARF (acute renal failure) b&. TEN (toxic epidermal necrolysis) ¢. AAC (antibiotic associated colitis) d. ADR (adverse drug reaction) 127. Which of the following is a major adverse effect of Chloramphenicol? a. ‘Thrombocytopenia b. Aplastic anemia . Hemolyticanemia 4. Agranulocytosis Reference Guide for For Questions and Answers- 2nd Revised Edition 128. Which of the following drugs mayNOT cause hemolysis in patients with G6PD deficiency? a Quinine b. —Sulfonamide c. —Nitrofurantoin d. Erythromycin 129. Ringing or buzzing in the ear is asso- ciated with : a, Streptomycin b. Salicylate c. Methyldopa 4. Minocycline 130. Pigmented retinopathy is highly asso- ciated with a. Chlorpromazine b. — Thioridazine ec. Clozaril 4. Haloperidol 131. Which of the following drugs is asso- ciated with pulmonary dysfunction ? 1. Bleomycin I. Amiodarone MI, Nitrofurantoin a Tonly b. Land Ionly cc. and If only dM, and [ff only 132. Which of the following phase trials includes extensive clinical tials ina human? I. Phase TIlonly TL. Phase I only I. PhaseTonly www.pharmacyexam.com ign Pharmacy Licensing Exam a Tonly b. —_TandIonly Hand Ilfonly 4. L.I,and only 133. When structurally different chemicals produce the same clinical results, itis generally defined as: ‘Chemical equivalence Bioequivalence ‘Therapeutic equivalence Clinical equivalence aoge 134, The parameter that is generally NOT included in evaluating the bioegivalency of ‘two or more formulations of the same drug is Peak height concentration. Concentration at receptor site. Time to reach peak concentration. AUC. apse 135. A substance that kills microorganisms but not bacterial spores is generally defined as: a. Bactericide b. Sterility c. — Germicide d. Disinfection 136. How many grams of Heparin is re- Aired to prepare I quart of 0.45% solution ? 2.21 grams 5.56 grams 4.32 grams 3.15 grams 137. Which of the following about moist heat sterilization is NOT TRUE? a Reference Guide for Foreign Pharmacy Licensing Exam Questions and Answers- 2nd Revised Edition a, _Itisawidely used method for sterl- ization of mineral oil, greases and waxes. b. The cause of death of organisms is generally attributed to coagulation of cellular proteins of organisms. c ‘The substance should be kept loose. to allow direct penetration of steam. d. The substance should be kept at least 15 minutes under pressure steam at a minimum of 12i° temperature. 138, Which of the following gases are gen- erally implied forsterilization of pharmaceu- ficals? 1 Bthylene oxide IL Formaldehyde I. — Sulfur dioxide a. Tonly b. Land I only c.— Tand Il only 4. 1, Hand {Ml only 139 How many ce of 75% alcohol is re- quired to prepare 35% of 500 cc of alcohol ? (You have 10% of 1000 cc of alcohol in stock.) a —-192.30ce b. — 200cc ce. 450ce d. 325.55ce 140. How many tablets (approximately) of 0.125 mg Lanoxin is required if the measured. plasma concentration of drug is 1 meg/ml and the desired plasma concentration of drug is 1.5 mcg/ ml. The apparent volume of digoxin is }OL/Kg for a 70 kg patient. (Assume S and F= 1) a One. b. Four ce. Three a Two www.pharmacyexam.com Krisman 141. Sterilization by U.V. radiation generally requires a wavelength of: a 320 nm b .253 nm e150 nm 4 475 nm 142. If i teaspoon of Thioridazine intense solution (30 mg/cc) is diluted up to the 480 ce mark with plain water, what would be the strength of the drug in mg/m) in the final so- lution ? a 1 mg fcc b. 0.52:mg /ec ©. 031 meee 4. 0.75 mgiee 143. Which of the following tests are used to identify Laminar flow efficiency and qual- ity of sterilization? 1 Smoke test I. Doptest Hil, Microbial test a. Tonly b. Land ITonly {and If only 4. LU and ttt only 144... The rate of hydrolysis depends on 1 Pressure I pHofthesolution T. — Temperature a. Tonly b. — LandITonly Mand Ilfonly 41, Hand itt only 22 Reference Guide for Foreign Pharmacy Licensing Exam Questions and Answers- 2nd Revised E 145. If 1000 iablets of Risperdal 1 mg cost ‘\— $2250, and the mark-up on prescriptions is 20%, ‘what would be the retail price of 30 tablets’? $150 $i7 $500 $81 ees 146. NH2 1H2 Ue —Q COOH ‘OH (P-Aminosalicylic acid) _(m-aminophenol) The formation of m-aminophenol occurs due to : a. Oxidation of p-aminosalicylic acid. b. _ Decarboxylation of p-aminosalicylic acid. c. Reduction of p-aminosalicylic acid. 4. Hydrolysis of p-aminosalicylic acid. 147. According to the International Phar- maceutical Federation, the maximum percent- age of overages is limited t0 : 10% of over the labeled potency. 20% of over the labeled potency. 30% of over the labeled potency. 40% of over the labeled potency. eos 148. At what concentration doessucrose re- tard the growth of microorganisms. = 20% wiw b. 30% wiw 65% wiw a 85% wiw www.pharmacyexam.com 149. Krisman ‘Which of the following best describes the Ingram’s regimen? a. Combination of anthralin + UVA b. Combination of coaltar + UVB cc. Combination of anthralin + UVB 4. Combination of psoralen + UVA 150. Which of the following drugs is indi- cated for treatment of eystic fibrosis? a. Domase Alfa b. Calcitonin salmon ©. Acetazolamide d. Methotrexate 151. Find out the ratio of ionized to union- ized species of a drug at pH =7. The pKa of the drug is 5 a 100:1 b.50:1 ce 25 a 200:1 152. Ocusert Pilo 20 generally delivers 20 meg of Pilocarpine per hour for 7 days. b. 20 mg of Pilocarpine per hour for 7 days. ¢. 20 meg of Pilocarpine per hour for3 days. 4. 20 mg of Pilocarpine per hour for 3 days. 153. Which of the following antiglaucoma medications has aCabonic Anhydrase Enzyme inhibition property? a. Timolol Db. —_ Dipiveftine 23 Reference Guide for Foreign Pharmacy Licensing Exam Questions and Answers- 2nd Revised Edition .— Dorzolamide 4. Latanoprost 4154, If a dropperiscalibrated to deliver 325 mg of iron sulfate per 0.6 cc, and the adult dose of the drugis 325 mg, what would be the dose of drug in cc required for a 15 month old infant . a Lec b. 03 cc c. 0.06 cc d 0.01 cc A155. Which of the following is/are true about Triphasic oral contraceptives? 1 Triphasic affects the follicular, ovula- tion and luteal phases of the menstrual cycle successfully and provides more favorable effects than biphasic and monophasic. TI Itis generally formulated with low progesterone content. TI. The principal advantage associated with Triphasic is the uniform direction for taking the pills. Tonly Land Il only Mandl only 1, Mand IJ only pege 156. Which of the following phase(s) is/are generally dominated by estrogen ? 1. Follicularphase I, Ovulatory phase IIL. Luteral phase a. Tonly b. —_TandIlonly www.pharmacyexam.com Krisman c. MandIIllonly 4. Mand Uf only 157. Which of the following about Metabolic Acidosis is NOT TRUE ? a. It generally occurs due to loss of bi- carbonate from the body. b. _ Hypokalemia is generally a result of metabolic acidosis. ¢. Administration of Arginine HCl gen- erally resolves metabolic acidosis. 4, It stimulates the respiratory center to increase excretion of CO, from the body. 158. Which of the following ingredients acts as a contact poison that disturbs the Parasite’s nervous system? a Lindane b, _Permethrine c, — Pyrethrin 4. Pyrimethamine 159, Which of the following is a cough ex- pectorant? a Guaifenesin b. —Dextromethorphan ¢. Benzonatate 4. Diphenhydramine 160, HbA, 6% value indicates : 80 mg/dl blood glucose 120 mgi/dl blood glucose 360 mg/dl blood glucose 160 mg/dl blood glucose Reference Guide for Foreign Pharmacy Licensing Exam. ‘Questions and Answers- 2nd Revised Edi 161. How many milliequivalents of Ferrous Sulfate are present in 325 mg of Ferrous sul- fate 2(Molecular weight = 151.85 gr/mole) 5.6meq 10.12 meq 7.5 meq 4.28 meq aoge 162. Which of the following adverse effect(s) is/are associated with insulin therapy? 1. Hypoglycemia IL _Lipoatrophy IIL. _Lipohypertrophy a Tonly b. and Monly ¢.— Tand Ill only 4. [Mand If only 163. Acarbose is contraindicated for pa- tients suffering from 1. Inflammatory bowel disease II Patients with colonic ulceration III. Patients with intestinal obstruction a Tonly b. Land ILonly cc. Tand Ill only @ [Mand Ill only 164. Which of the following isNOT asymp- tomof hypoglycemia? Confusion Bradycardia Difficulty in concentration ‘Sweating pose 165. Whichofthe following diabetic drugs pro- duces disulfiram reaction with alcohol? www.pharmacyexam.com Krisman a. Acarbose b. _Chlorpropamide cc. Metformin d.—_Repaglinide 166. Normal threshold value for glucose inthe kidney is a. 100 mg/dl b. 120 mg/dl cc. 180 mg/dl 4220 mg/dl 167. How many mEqs of Na’ are present in \/ 0.9% 250 ce solution of NaCl ? {equivalent weight of NaCl = 58.5] a 50.5 mEq b. 38.5 mEq c. 225 meq d. 585 mEq 168. Which of the following is NOT rec- ‘ommended to treat hypoglycemia induced by Acarbose ? Dextrose Table sugar Glucagon Glucose eer 169. | Whatis the half-life of a drug that has a rate constant of 0.067 days” in first order kinetic ? a Sdays b.-7.23days ©. 10.34 days 4. 12.51 days 170. Which of the following isNOT TRUE about Vancomycin HCl? 25 Reference Guide for Foreign Pharmacy Licensing Exam Questions and Answers- 2nd Revised Edition a, Nephrotoxicity and ototoxicity are ‘comtian complications of therapy, b. IV Vancomycin is indicated for treat mentof antibiotic associated coli c. When taken orally, itis poorly absorbed from the GIF. 4. Ttisavailable in oral solution, pulvules 4nd injection form, 171. Which of the following drugs shoutd patients with hypertensensitivity to Me- salamine avoid ? 1 Sulfasalazine TL — Olsalazine It. Mesalamine a, Tonly b, Land only ©, Mand If only 4 Eitand {ff only 172, Which of the following is NOT a sign of Grave’s disease? 4 Fast heartbeat b, Weight gain a Soft skin d. High basal metabolic rate 173, Which of the following is/are caused by thyroid hormone deficiency ? 1. Myxedema M— Cretinism Ml, Grave's disease a. Tonly b. Land Il only © Mand Ilfonly 4 I,Mand Mf only ‘www.pharmacyexam.com Krisman 174, Which of the fottowing thyroid drugs should in pregnant women avoid ? Propylthiouracil Methimazole Propranolol Levothyroxine eee 175, Which of the following statements about thyroid supplements islare true? I. _Livothyronine is associated with head- ache, palpitation, tremor and diarrhea, and is less recommended for treatment of hypothyroidism. I. Desiccated thyroid preparations have a variable T3 to T4 ration and are less recommended by physicians. IIL Levothyroxine is the most recom- mended thyroid supplements. Tonly Land IT only Mand I only J, Wand Wi onty eo se 176. Which of the following drugs is indi- cated in smoking cessation programs ? a. Bupropion b. Tramadol cc. Tamsulosin 4. Risperidone 177. Which of the following drugs inhibits the aggregation of platelets ? 1. Ticlopidine HCL IL Aspirin Il. Clopidrogel Reference Guide for Foreign Pharmacy Licensing Exam Questions and Answers- 2nd Revised Edition a Tonly b. Land Ionly c. Hand Mfonly d. I, Mand If only 178. Which of the following drugs mobilizes bone calcium into blood ? Estrogen Calcitriol Calcitonin Salmon Alendronate Na apse 179. Diphydrotachysterol is indicated for treatment of 1 Tetany I Idiopathic Tetany II. Hypoparathyroidism Tonly Land I only Tand Il only I, Mand III only aege 180. Which of the following is NOT a sign of Morphine overdose? a, Cold and clammy skin b. Respiratory depression c. Pupil dilation 4. Convulsions 81. How long will it take to decompose Lanoxin in elixir to one half of its original concentration in an order that is first order kinetic ? [ K = 0.023 min} a. 40.45 minutes b. 50.60 minutes c. 75.80 minutes d. 30.13 minutes 182. Which of the following glasses are most chemically resistant or least leachable? www.pharmacyexam.com Krisman a, Type borosilicate glass b. Type ITsodalime treated glass cc. Type III sodalime glass d.—TypelVglass 183. According to Federal standard, class 100 clean room is generally defined as a, Not more than 100 people working in room. b. Not more than 100 particles per cubic ft of 0.5 mom or larger size. c. Not more than 100 bacteria per cubic ft of 40 mem or larger size. 4. Not more than 100 viruses per cubic ft of 40 mem or larger size. 184. Which of the following isNOT TRUE about a HEPA filter ? a. It is generally described as a high effi- ciency particulate air filter. b. thas anefficiency of removing 100% of particles 0.3 mem or larger. c. A Dioctyl Phthalate test (DOP) generally finds outthe efficiency of HEPA filters. 4. __Itisuseful forthe preparation of parenteral products. 185. Cold sterilization of parenteral solution is defined as: a, Removing 2 memorlarger par- ticles from the solution. b. Removing 0.2 mem or larger par- ticles, including microorganisms, from the solution. Pa Reference Guide for Foreign Pharmacy Licensing Exam Questions and Answers- 2nd Revised Edition ©. Sterilization of the parenteral solution with hot and cold compresses. 4. Sterilization of he parenteral solution by freezing the solution below freezing point 186. Lyophilization is defined as: 2. The removal of water by method of sublimation from the product after it is frozen. b. Direct evaporation of a substance from its solid form. c. Preparation of colloidal solution, 4. Removat of particulate matter by fil- tration. 187. Which of the following is NOT de- scribed as aquality assurance and control test for parenteral solution? Sterility test Particulate matter evaluation Pyrogen test Schilling test pegs 188. Which of the following isNOT anon verbal communication? a. Body posture b. Facial expression ¢. Open-ended question 4. Distance of the patient 189, Staff model HMO organizations gen- erally own: L Healthcare facilities TI. Employed physicians II, Nonemployed physicians www.pharmacyexam.com a. Tonly b.— Tand only ¢. Hand!lfonly 4 [Mand ifonly 190. ‘The disadvantage associated with mail- order pharmacy services are : L Alack of pharmacist counseling. I, Allack of review of patient’s pro- file. II, ‘The use of recycled prescription drugs. a Lonly b. Land II only Wand {fl only 4 I, Tand Hit only 191. Which of the following best describes the prospective payment system? a. Reimbursement to the hospital based on cost incurred during hospitalization. b. Reimbursement o the hospital in ad- vance, with a specific rate based upon diagnosis of patient. ¢. Reimbursement to hospital by patient when he/she i discharged. 4. Prescription purchased by patient by paying cashs. 192. Most hospital pharmacies generally 401. Liquidity generally expresses a pharmacy’s ability to meet its: a Assets b. Current liability c. Inventory d. Prepaid expenses 402. The acid test generally measures a pharmacy’s Financial position Liquidity Profitability Inventory pose 403, Which of the following is generallyNOT included in current assets ? a Cash b. Accounts payable Accounts receivable 4. Inventory 404, Which of the following would generally be considered the fixed assets of a pharmacy? 52 Reference Guide for Foreign Pharmacy Licensing Exam Krisman (Questions and Answers- 2nd Revised Edition a Inventory D LONG TERMLIABILITY b. —_Fixturesand equipment c Ch Notes payable (>1 yr) $20,000 d. Accounts receivable Total liabilities $105,000 Net worth $ 165,000 405. Allofthe following can be considered the Cost of goods sold $490,000 current liability of a pharmacy EXCEPT a, Accounts payable. b. Notes payable within 1 year. c. Accrued expenses. d. Notes payabie beyond 1 year. 406. Find out the Acid test (quick ratio) of Manancare Pharmacy from Table 1? a 202/1 BLA ef com TABLE 1 A CURRENT ASSETS Cash $50,000 AIC receivable $75,000 Inventory $ 100,000 Prepaid expenses $ 10,000 Total current assets $235,000 B FIXED ASSETS Fixtures and equipment $ 30,000 Deposits $5000 Total fixed assets $35,000 TOTAL ASSETS $270,000 C CURRENT LIABILITIES AIC payable $70,000 Notes payable (1 yr) $5,000 Accrued expenses $ 10,000 Total current liabilities $85,000 www.pharmacyexam.com 407. Which of the following does NOT measure the pharmacy’s liquidity ? Acid test ratio Current ratio Net sales to inventory Inventory to its net working capital pers 408, Total liabilities to net worth ratio of Manancare Pharmacy is : a. Acceptable b. Below expectation c. Exceeds the expectation 4 Cannot be calculated 409. The investment in fixed assets of MananCare Pharmacy : Exceeds the requirement Is below the requirement Meet’s the requirement Cannot be calculated pegs 410. Manancare Pharmacy wants to sel its pre- scription files. The Manancare Pharmacy own- cers asks $350,000 for the existing prescription file. ‘The Manancare Pharmacy provides the follow- ing data upon request. Total new RX dispensed in past 2 years. $ 80,000 The %ofRxthathasone 40% or more refill left Reference Guide for Foreign Pharmacy Licensing Exam Questions and Answers- 2nd Revised Edition The average RX price $50 Net profit % % 15 What would be youranswer to the owner of the pharmacy? Price is okay. Price is too high. Price is breaking even. Cannot be calculated. aoge 411. “Manancare Pharmacy” marksdown the price of analgesic balm from $3 to $2. If the mark down of the price increases the sales of analgesic balm from 60 tubes f0 80 tubes, what would be the coefficient of elasticity ofthis prod- uct? aod b 2 ce 025 a 05S 412, When relative change in revenue is same as the relative change in price, itis known as: 1 Unitary elasticity Inelastic demand Elastic demand Tonly Tand Ionly Mand IIT only 1, Iand II only a b. c. 4 413, Find out the retail price of abox of insulin syringes ifthe cost complement ofthe productis 55% and the cost of one box of insulin is $9.00. a $4.95 b. $16.30 ee Sis ad $13.95 www.pharmacyexam.com Krisman 414, Find out the % markup of Vasotec pre- scription if 30 tablets of Vasotec 5 mg retail price is $75 and the cost of the drug is $45 a oe b. 15% 66% d 10% 415, Find out the retail price of one box of insulin syringes if The cost of complement 55% The known retail markup = 45% The cost of syringes $9.00 a 495 b. 13.95 c 16.30 a isi 416. For Manancare Pharmacy, the total rent for the whole store including the Phar- macy department is $70,000. The size of the phar- macy is 600 fé and the size of the whole store is 5000 fe? . On the basis of above figures, what ‘would be the rent of the pharmacy alone? $1000 $2000 $1200 $800 aes 417. The funding for Medicare programs is generally obtained from: TL. —_Social security taxes TL Premiums paid by participant TI. State government a Tonly b. Land Ionly ¢. Tand Ill only 4. I,M and ff only Reference Guide for Foreign Pharmacy Licensing Exam Questions and Answers- 2nd Revised Edition 418. Ina patient cost sharing plan, when a patient has to pay a specified amount of the cost of prescriptions and a third party will pay the remainding cost of prescriptions, it isknown as: a. Copayment b. Coinsurance c. Deductible d. Retrospective payment 419. A person who works for an insurance company, provides the statistical data that in- dicates the risk associated with serving the popu- ‘ation, and determines the premiums to cover all the estimated expenses is known as: a. Pharmacy manager b. Actuary Sponsor 4 Vendor 420. ‘The maximum amount that will be paid by a third party to a pharmacy when the drug isavailable from more than one source is defined as ‘Maximum allowable cost (MAC) Estimated acquisition cost (EAC) Actual acquisition cost (AAC) Average wholesale price (AWC) aege 421. When a patient pays a full predeter- mined amount to the provider at the begin- ning of each month it isknown as: a. Concurrentreimbursement b. Prospective reimbursement c. Retrospective reimbursement 4. Cashreimbursement 422. The increase in the number of taking the foreign pharmacy exam isas follows: www.pharmacyexam.com Krisman Year Students taking the exam. 1981 350 1982 420 1983 530 1984 600 1985 620 1986 635 1987 700 1988 850 Find out the mean of the above data: a 601 bd. 588 ce. 720 a 520 423. What would be the median of the above example? a 35 b. 850 c 610 a 635 424. A random sample of the blood glucose “concentration of 100 patients has a mean of 130 and a median of 155. The frequency dis- tribution of the sample is: Normally distributed Positively skewed Negatively skewed Cannot be calculated pose 425. Allof the following can be a shape of frequency of distribution EXCEPT: a. Bell shaped distribution b. —_Skewed shape distribution cc. Ushape distribution d. TT shape distribution 55 Reference Guide for Foreign Pharmacy Licensing Exam Questions and Answers- 2nd Revised Edition 426. What would be the Pearsonian coefficient of skewness if.a sample has a mean of 55 anda median of 45. The standard deviation of the sample is 35. 0.90 1.0 0.85 035 ae ee 427. Which of the following about a Bino- mial experiment is NOT true? a. Each trial results in an outcome that is classified as success or failure. b. _Therepeated trials are dependent upon. previous experiment. c. The experiment generally consists of n-repeated trials. d. The probability of success remains ‘constant from trial to trial. }28. Whatis the mean binomial distribution if the probability of success is 0.60 in 50 trials ? a5 B38 . 8 a4 429. If the blood pressure measurement of 5 people is 110, 135, 140, 125 and 115. re- spectively . What would be the range of the set of the above observations? a0 vb. 30 c 125 a 140 430. When plotting t distribution curves, if sample size of 20s taken from a normal popula- tion, what would be the degree of freedom in the tdistribution? www-pharmacyexam.com a 40 bie «10 a 2 1. Find out the degree of freedom ina 2x3 contingency table Chi-square test when it is applied to test the hypothesis of independence of two variables? ees Hane 432. The average length of time it takes stu- dents to finish an exam is 180 minutes, with a standard deviation of 36 minutes. A new ex- amination procedure using modern comput- ers is being tested. A random sample of 50 students had an average examination time of 150 minutes, with a standard deviation of 40 minutes under the new system. Test the hypothesis that the population mean is now less than. 180 minutes. This hypothesis would resultin ? a. Onesided b. Twosided c Three sided da. ‘Cannot be calculated 433. In protein, Amino acids are joined co- valently by: Hydrogen bond Peptide bond Oxygenbond Disulfide bond pose 434. The secondary structure of protein consists of : 1 Alfa-helix I. Beta-sheet Ti —Beta-bend Reference Guide for Foreign Pharmacy Licensing Exam Questions and Answers- 2nd Revised Edition Tonly Tand I only Tand IIT only 1,1, and I only pose 435. The denaturation of protein can occur in the presence of : lL Heat I Strong acid TI. Organic solvent Tonly Tand Il only Wand ff only I, and III only eege 436. Which of the following about sickle cell anemiaisNOT true? Itisa genetic disorder resulting from the production of a variant hemoglo- bin, b. _Itischaracterized with pain, lifelong hemolytic anemia and tissue hypoxia. ¢. Thereplacement of leucine at the sixth position of the Beta-globulin chain for glutamate is generally responsible for causingit. d. The form Hbshas an extremely low solubility compared to Hba which results into the aggregation of ‘molecules that form or create sickle shaped red blood cells. 437. The enzyme with its cofactor is generally knownas: Coenzyme Holoenzyme ‘Apoenzyme Prosthetic group aege www.pharmacyexam.com 438. The process in which the release of en- ergy from energy rich molecules such as glucose and fatty acid occurs in mitochondria is known = a. Oxidative decarboxylation b. Oxidative phosphorylation ©. Oxidative deamination 4. Oxidative dehydrogenation 439. The breakdown of complex molecules such as protein, lipid and polysaccharide into simple molecules such as carbon dioxide, water and ammonia is known as: a. Aerobic glycolysis b. —_Catabolie reaction ©. Anabolicreaction 4. Gluconeogenesis 440. What would be the end product of gly- colysis in the cell with mitochondria ? a Glucose b. Glycogen ©. Pyruvate d. Lactate 441. Which of the following substance lev- els is found to be deficient in a patient with G6PD deficiency? a. Alfa-antitrypsin in reduced form b. —_Bradykininin reduced form ¢. Glutathione inreduced form 4. Trypsininelevated form 442. Which of the following causes hemolytic anemia in patients with G6PD deficiency? 1. Oxidantdrug IL __ Ingestion of fava beans TI —_Certaintypes of infections 57 Reference Guide for Foreign Pharmacy Licensing Exam Questions and Answers- 2nd Revised Edition a Tonly b. Land I only ¢. — Tand Ill only 4.1, Mand IIfonly 443. Which of the foliowing should be classi- fied as a disaccharide? a Ribose b. Lactose ©. Glycoprotein 4. Glycosaminoglycans 444. The pairs of structure that are mirror images of each other are known as: a Isomers b. —_Enantiomers ce. — Epimers 4. Muta rotation 445. In humans, the principle storage of glycogen is found in the : u Skeletal muscle IL Liver 1. Spleen a Tonly b.— Tanditonly cc. Handilfonly 4 [,Mandifonly 446. Which of the following substances should be classified as a polysaccharide? a. Glucose b. Hyaluronic acid c. Sucrose d. Glycoprotein 447. Which of the following agents acts as an emulsifying agent for metabolism of lipid in duodenum? www.pharmacyexam.com 2. Gastric lipase b. Bile salt c. Pancreatic juice Secretion 448, Whichof the following aboutsteatorrhea isNOT'TRUE? a. Tteauses alossof lipid, essential fatty acid and lipid soluble vitamin in fe- ces. b. The oversecretion of bile salt may im- pair the absorption of fat soluble vita- mins. ¢. The inhibition of secretion of pancre- atic juice from the pancreas generally results in steatorrhea 4d. _ Theabsorptionof vitamins suchas vitamin C, thiamine, and riboflavin are not affected by the condition of steatorrhea. 449. Which of the following is a building block of membrane of nerve tissue? a, Prostaglandin b. —Spingomyetin c. Thromboxane 4. Leukotriene 450. Thetermcholelthiasisis generally refered toas: a, Obstruction of the stomach by choles- terol stone. b. Obstruction of the pancreas by choles- terol stone. cc. Obstruction of the gall biadder by cho- lesterol stone. 58 Reference Guide for Foreign Pharmacy Licensing Exam ‘Questions and Answers- 2nd Revised Edition Obstruction of the spleen by cholesterol stone. 451. Which of the following isNOT a func- tion of Luteinizing hormone? a Itinitiates the testosterone synthesis in the Leydig cells ofthe testis. b. It simulates the process of spermatogenesis, ¢. _ Itinduces ovulation in females. d. __Itstimulates synthesis of progesterone and estrogen in the corpus luteum, 452. Thetotalenergy required by an individual can be found by calculating : 1. BMR ‘Thermic effect of food MI. Physical activity Tonly Tand Il only Mand II only I, Mand Ill only eore }53. Which of the following is considered pro- tein deficient malnutrition? 1. —_Kwashiorkor TL Marasmus Steatorthea a Tonly b. Land ITonly ©, Mand Ifonly aI, Mand Il only 454, Which of the following is NOT a water soluble vitamin? www.pharmacyexam.com Krisman a Thiamine b. Vitamin c. Niacin d. Pyridoxine 455. Which of the followings a good source of Vitamin K? Cabbage and cauliflower Fatty fish and liver Vegetable oils Yellow and green vegetables and fruit aege 456. ‘Theend product of purine catabolism is: Alantoin Uric acid Hypoxanthine Xanthine aoege 457. Thesmall andcircularextrachromosomal DNA molecules that carry genetic information for future generations in bacteria are known as: a. Lysozymes b. Plasmids = Mitochondria d.— Cytoplasm 458. Which of the following RNA types comprises 80% of the RNA in the cell? IL. Ribosomal RNA I. Transfer RNA II. Messenger RNA Toniy Land If only Mand {tf only 1, Hand Ill only eoge 459. Which of the following types of RNA carries genetic information from DNA to cy- tosol for protein synthesis ? 59 Reference Guide for Foreign Pharmacy Licensing Exam ‘Questions and Answers- 2nd Revised Edition 1 mRNA 1 tRNA Ml rRNA a. Tonly b. Land I only c. Hand Ilfonly 4. L,Iand Ilonly 460. Which of the following codons are generally known as stop or nonsense codons? I UAG Tl, UGA Tl. UAA a Tonly b. — TandIonly c. Mand Mf only da I, land Ill only 461. Which of the following types of micros- copy is used to observe the unstained living or C0, + CO. B The addition of a 5 liter solution of Catcium oxide in the above mixture would change the re- actionin which direction? a. Direction b. Direction B c Remains same 4. Cannotbe predicted 548, Which of the following is the correct the ‘Way of expressing strength of the following acids in descending order ? H,CO, >HCIO, >CH,OH >NH,’ NH, >HCIO, > H,CO, > CH,OH HCIO, > H,CO, > NH,’ > CH,OH CH,OH > NH, > HCO, > H,CO, ae oe 549. Find out the Ksp of Li,PO, (0.5 gm/ _¥ 100 ce) solution. a 5x10° b. — 9.22x 10" c 4x10" d 25x 10° 550. Thechlorination of Nitrobenzene gener- ally results in a. Ortho Chioronitsobenzene b. Para Chloronitrobenzene ¢. -MetaChloronitrobenzene 4. Chlorobenzene 551. The nitration of benzaldehyde will result in: a. Metaisomer b. Ortho isomer c.— Paraisomer d. Ortho and para isomers 69 Reference Guide for Foreign Pharmacy Licensing Exam ‘Questions and Answers- 2nd Revised Edition 552. 2 Methyl 2 Propranolol is classified as a: a. Primary alcohol b. Secondary alcohol c. Tertiary alcohol d.__ Polyhydroxy alcohol 553, Whatisthe IUPAC name of the follow- ing compound? cS cH So? i ° a. Benzylphenone b. Acetyl phenyl ketone c. Acetyl benzyl d. — Propranone 554. The oxidation of secondary alcohol generally results in: a Aldehyde b. Acid ce. Ketone d. Primary alcohol 555. The IUPAC name of the following compoundis: cH, ¢ -CH,-COOH a Beta chlorobutyric acid Alfa- chforobutanoic acid Alfa- chloropropanoic acid 2 chloropropanoic acid ence 556. Which of the following compounds has the lowest PKa value? Methylamine Ammonia Aniline Methylethylamine eese ‘www.pharmacyexam.com Krisman ,-557. Which the following compounds pro- duces a high concentration of H,0* ions when dissolved in H,O? a, P-nitrophenol = 2,4,6 trinitrophenol c Phenol 4. Ortho nitro phenol 558. The risk of teratogenesis isthe high- estduring which trimester ? a Firsttrimester b. Second trimester ¢. Third trimester Fourth trimester 559. Which of the following categories has shown the highest risk to the fetus? a. Category A b. Category B c. Category C a Category X 1-560, Which ofthe following antibioticsis the drug of choice for treatment of UTI in pregnant ‘women in their first trimester? a. Tetracycline b. SMZ/TMP ciprofloxacin 4. Metronidazole 561. Allofthe following drugs are teratogenic in nature and should be avoided by pregnant women EXCEPT: a, Finasteride b. —Triretinoine cc. — Benztropine 4 Lithium carbonate 0 Reference Guide for Foreign Pharmacy Licensing Exam Questions and Answers- 2nd Revised Edition 562. Which of the following are essential fatty acids in humans? 1 Linoleic acid I. Linolenic acid Il. Arachidonic acid only Tand Ionly Mand III only 1, Iand Ill only aeoe 563. Ifapatient with 30% of hematocrit lost 1000 ce of blood in a car accident, what would be the dose of iron dextran (50 mg/ml) to re- place the iron loss ? a oak b 6 mi cin a 2m 564. Theprincipalenergy-carrying molecule in acellsis: a AMP b. ADP c. AP d NADP 565. Whichofthe following drugscan be used as an adjunctive therapy with Coumadin in a Patient with cardiac valve replacement? a. Lidocaine b. —_Dipyridamole ~ percestnt- c. — Phenytoin 4. Diazepam 566. Which of the following parenteral routes should be avoided with heparin? a SC bo iM: www.pharmacyexam.com Krisman GS d.—_Inermittent IV. 567. Whichof the following tests is widely used tomonitor heparin therapy ? PIT APTT Whole blood clotting time INR aere 568. Allof the following would increase the risk of bleeding with Coumadin therapy EXCEPT: a. Cefamandole b. Cimetidine cc. Phenobarbital 4. Chlorpropamide 569. Which of the following drugs is used for treatment of hemophilia? a. Aminocaproic acid b. Hemin c. _ Antihemophilic factor a Estrogen 570. Which of the following mechanical contraceptives has shown the least risk of preg- nancy? a. Diaphragm b. Vaginal sponge Condoms 4 UD 571. Rank the following glucocorticoids according to their expected half lives: a, Cortisone > Triamcinolone > Betamethasone > Hydrocortisone b. __Betamethasone > Triamcinolone> Hydrocortisone > Cortisone mn Reference Guide for Foreign Pharmacy Licensing Exam ‘Questions and Answers- 2nd Revised Edition ¢. Triameinolone> Hydrocortisone > Cortisone > Betamethasone 4. Hydrocortisone > Betamethasone> Cortisone > Triameinolone 572. Which of the following corticosteroids has the highest mineralocorticoid activity ? a. Prednisone b. —_ Methylprednisolone c. Dexamethasone d. — Fludrocortisone 573. Aminoglutethimide is it treatmentof: a. Bleeding disorder b. Cushing's syndrome c. Hemophilia d. Prostatic cancer 574, Rank the following insulins in de- scending order according to their duration of action: a. Prompt Insulin Zn > Protamine Zn > Tsophane Insulin Zn > Insulin lispro b. _Protamine Zn Susp> Isophane insulin Protamine Zn insulin > Insulin lispro ¢. Insulin lispro > Protamine Zn > Prompt Zn > Isophane insulin 4. Protamine Zn> Prompt Zn> Insulin lispro> Isophane 575. Which of the following sources of insu- linsis the least antigenic in nature ? a. Sheep b. Beef c. Human 4. Pork www.pharmacyexam.com Krisman 576. “Insulin resistant” is defined as when a patient needs more than how many units of insulin per day? ic 80, ibe 120 c. iso dG 200 577. Which of the following symptoms helps to differentiate diabetic ketoacidosis from hypoglycemia? I. Rapidpulse. II. The presence of glucose and acetone in urine. I. Acetone breath. a. Tonly b. Land Ionly c, Mand [only d. I, Mand If only 578, Thyroid 1/4 grains is equivalentto: a 16 mg b. 32 mg c 65mg 4. 8 mg 579. What would be the volume of 5% potas- sium iodine that is required to mix with a 10% solution of potassium iodide to make a7.5%, 500 cc solution of potassium iodide ? a 280 ce b. 500 ce & 150 ce d 200 cc 580. Which of the following diuretics acts by inhibition of Na’K*ATP-ase activity? nR Reference Guide for Foreign Pharmacy Licensing Exam Questions and Answers- 2nd Revised Edition a Tiiamterene b. Spironolactone ©. Amiloride 4. Furosemide 581. How much Mannitol is present in 150 cc of 15% solution of Mannitol ? 15.0 gm 22.5 gm 44.0 em 75 gm eee 582. If 75 ml of 25% Mannitol solution is diluted up to 800 cc with SWFI, what would be the percentage of the final concentration of Mannitol ? a 234% So e G6 @ d 234% 583. Lanoxin (digoxin) can be indicated for the treatment of all of the following EXCEPT. a Atrial flutter b. Atrial filtration c. Ventricular fibrillation d CHF 584, If apatient is taking Lanoxin (digoxin) for treatment of CHF and is then diagnosed with myxedema, the dose of the drug should be: Kept the same Reduced and monitored Increased and monitored Omitted aoge 585. Which of the following is tue about the toxicity associated with Digoxin ? www.pharmacyexam.com Krisman 1. Avoid the use of potassium supple- ‘ments in patients with complete heart block because of Digoxin. II, Severe sinus bradycardia can be treated with atropine. TI. Forcomrection of asthythmia, lidocaine or phenytoin can be used. Tonly Tand only Wand IM only 1, Mand IM only pesos ‘586. Which of the following adverse effects is commonly reported with the prolonged use of, Amrinone? a SLE b. Thrombocytopenia c. granulocytosis 4. Peripheral neuropathy 587. Rank the following Nitroglycerine prod- ‘ucts according to their onset of action: a. Nitrostat SL> Nitrolingual spray >Nitrostat IV > Nitrodur TDS b. —_Nitrostat LV > Nitrostat SL > Nitrolingual spray > Nitrodur DS ¢. _Nitrolingual spray > Nitrostat IV > Nitrostat SL> Nitro TDS d. —Nitrostat 1V >Nitrolingual spray > Nitrostat SL > Nitrodur TDS 588. Whichofthe followingis the correct way of administering Nitroglycerine at the frst sign of, ‘cute anginal attack ? 3 Reference Guide for Foreign Pharmacy Licensing Exam Krisman Questions and Answers- 2nd Revised Edition Dissolve two tablets under tongue and. $92. The use of Procainamide is contraindi- repeat steps every 30 mins until pain cated in patients with: iscontrolled. a Amhythmia b. Dissolve one tablet under the tongue b. Myasthenia gravis at the onset of attack and repeat every c. Ulcerative colitis 4hours for2 days. 4. Zollinger-lision syndrome ¢. Dissolve one tablet under tongue at 593. Which of the following drugs may pre- acute onset of attack, repeat every 5 cipitate Lupus erythematosus-like syndrome? ‘minutes until complete reliefs achieved, Do not exceed more than 3 1 Procainamide tablets withina 15 minute interval I. Hydralazine Ti. Lidocaine 4. Dissolveone tablet under tongue at on- set of attack and prepare patient for A fey administration of LLV. Nitoglycesins b. Land Ionly c. TL and Il only 589. The Cardiotoxic effects of quinidine can d 4,0 and Ill only be reduced by administering : 594. Patients with congestive hear failure have a, Sodium bicarbonate to avoid which of the following? b. —Phenytoin c. Sodium lactate a, Procainamide d. Diazepam b. —_Phenytoin c. Disopyramide 590. Ringingin theear, headache, nausea, ver- d. Lidocaine tigo, and vision disturbances are reported side effects of: 595. Which of the following are reported side effects of Lidocaine ? a, Procainamide b.Phenytoin 1 Malignant hyperthermia ec. Quimidine I. —CNStoxicity d.— Flecainide TI, Hepatic failure 591. Which ofthe following salts of Quinidine a Tonly contains the highest amount of anhydrous quini- b. Land Ionly dine alkaloid ? c I and III only d. 1,1 and IM only 4. Quinidine sulfate b. —— Quinidine gluconate 596. Which of the following classes of antiar- ¢. Quinidine polygalacturonate rhythmic agents is restricted only for life- 4. Quinidine hydrochloride threatening arrhythmia? www.pharmacyexam.com 14 Krisman Reference Guide for Foreign Pharmacy Lice Questions and Answers- 2nd Revised Edition a Class TA. b. Class IB c.— ClassIC 4. Class Ill 597. Which of the following potassium salts contains the highest meq amount of potassium? a Potassium chloride b. Potassium bicarbonate ©. Potassium citrate d. Potassium gluconate 598. Whichofthe following vitamin A deriva- tives helps with visual adaptation in darkness? a. Retinol b. Beta Carotene c. Retinoic acid 4. Retinal 599. Which of the following are precur- sors of vitamin D? L__Exgosterol I. 7-dehydrocholesterol Dihydrotachysterol Tonly Land II only Mand Hf only 1, Mand If only a, b. c a. 600. Which of the following calcium salts has the lowest amount of elementary calcium? a. Calcium carbonate b. Calcium gluconate c. ‘Calcium citrate 4. Calciumacetate 601. Urinary alkanizers may increase the excretion of all of the following drugs EX- CEPT: www.pharmacyexam.com a. Methotrexate b.— Quinidine c. Salicylate 4. Methenamine 602. Which of te following drugs is indicated for the removal of excess K* ions from blood? a. Calcium phosphate b. EDTA c. Sodium polystyrene sulfonate d.— Deferoxamine 603. Which of the following drugs indicated san appetite stimulant agent? a. Epotein b. — Megestrol acetate c. Pyridoxine d. Ensure 604. Which ofthe following drugsis indicated for the treatment of Benign prostatic hyperplasia? a Danazole b. Finasteride cc. _ Medroxy progesterone Mannitol 605. Which of the following is a principal adverse effect of anabolic steroids? a, Hypercholesterolemia b. Peliosis hepatis ce Amenorrhoea_ da Anuria 606. Prostate cancer is known to be sensitive Progesterone Diphydotestosterone Estrogen Follitropine eege 18 607. Oxytocin generally causes: Dilation of uterine muscles. Dilation of coronary arteries. Contraction of uterine muscles. Contraction of prostate gland. eee 608. Which of the following drugs is/are indicated for treatment of Paget's disease of bone? L Alendronate sodiun I. Etidronate disodium TI. Tiludronate sodium a Tonly band Ionly cc. — TMand Ill only 41, Mand Hf only 609, Which of the following insulins has a rapid onset of action and the shortest duration of action? Regularinsulin Prompt insulin Zn Insulin lispro Insulin Zn suspension eege 610. Which of the following Sulfony! urea agents should be avoided by geratic patients ? ‘Tolbutamide Chlorpropamide Glyburide Glipizide pose G11. Table sugaras source of glucose for the treatment of hypoglycemia should not be used by patients with : a. Metformin b. — Acarbose cc. Glyburide 4 Insulin www.pharmacyexam.com Pharmacy Licensing Exam Questions and Answers- 2nd Revised Edition 612. Which of the following diabetic agents has the greatest risk of hepatotoxicity? a. Metformin b. Troglitazone cc. Glyburide @—Insulin 613. Patients with hypersensitivity to thiazide diuretics may need to avoid: 1. Diazoxide I Metolazone Tl, Hydrochlorothiazide a. Tonly b. Land I only ce. Hand Ilfonly 4 1, 1and Ill only 614. Which of the following antihyperthy- roid agents.can be safely administered to preg- nant women ? Methimazole Propylthiouracil Potassium iodide Levothyroxine eese 615. Which of the following isa therapeutic use of Desmopressine? Diabetes insipidus I. Hemophilia I Williebrand’s disease a. Tonly b. Land Ionly cand Illonly 4. L,I and IMfonly 616. Octerotide acetate generally inhibits the secretion of : 16 Reference Guide for Foreign Pharmacy Licensing Exam Questions and Answers- 2nd Revised Edition ‘Thyroid hormone Growth hormone Nor adrenaline Serotonin pegs 617. Whichof the following drugs is indicated for treatment of hyperammonemia? Lactulose TL Sodium benzoate and sodium phenylacetate II. — Methamphetamine Lonly Tand only Mand I only 1, ITand I only Boge 618. Prolonged treatment with Bromocrip- tine should be monitored through: a. Retina function b. Pulmonary function ¢. Renal function 4. Hepatic funetion 619. Probenecid may elevate the serum con- centration of which of the following drugs? I Methotrexate UL. — Retrovir I. Pantothenic acid. a Tonly b. Tand I only c. Hand llonly d. 1, Tand ik only 620. Cyanide poisoning can be treated with: Deferoxamine Mesna Diazepam Asnylnitrite eoge www.pharmacyexam.com Krisman 621. Whichof the following drugs should be used for treatment of Cyclophosphamide induced Heeystitis? a. Dimercaptrol b. — Leucovorin Ca ©. Mesna 4 Vitamin K 622. Which of the following drugs antagonizes ‘the action of opioid drugs? 1. Naloxone Il Nalmefene Th Naltrexon a Tonly b. Land Ionly Mand Sif only 4 I, Iand If only 623, Which of the following drugs antago- nizes the toxic effect of benzodiazepine over- dose ? a. Deferoxamine b. Fluoxetine c. Flumazer 4. Chlorpromazine 624, Which of the following informations is NOT true about activated charcoal ? a. Itisindicated for reatmentof drug intoxication in an unconscious patient. b. __Itisacarbon residue derived from or- ganic material by exposing it to ant oxidizing gas compound of steam, oxygen and acid at a high temperature. Activation of charcoal surfacesin- creases the adsorption properties. 7 Reference Guide for Foreign Pharmacy Licensing Exam Questions and Answers- 2nd Revised Edition Bach gram of charcoal is capable of bind 100 mg to 1000 mg of drug. 625. Alkalinization of urine increases the bioavailability of : a. Propranolol b. Verapamil c.— Cimetidine d.— Flecainide 626. Whichof the following Beta-blockers has Beta-I receptor selectivity ? a, Pindolol b. Sotalol €. Propranolol 4. Acebutolol 627. A positive Comb’s Test helps to identify hemolytic anemia associated with: a. Chloramphenicol b. — Procainamide c. Methyldopa d.— Flecainide 628. Which of the following tests isem- ployed to evaluate tablet dosage form? 1. Weight variation I. Disintegration TM Dissolution a. Tonly b. Land only Mand {lfonly 41, Mand If only 629. Breakdown of tablets into smaller par- ticles or granules is defined as the process of: a. Disintegration b. Dissolution www.pharmacyexam.com Krisman c.— Rupturization 4. Granulation 630, Which of the following is/are pro- cessing problems for a manufacturer of tablets ? 1. Capping Tl Mottling Ml. Picking a Tonly b. Land Honly ¢. Tand Blonly 4. [land Iifonly 631. Which of the following ingredients used as a disintegrant for tablet dosage form ? a. Starch b. Lactose coe Tac d.— Stearicacid 632. Which of the following ingredients increases the flow property of granules ? a Acacia b. Comstarch c Clays Mannitol 633. Which of the following diluents should be avoided with Tetracycline ? a Lactose b.— Mannitol cc. Calcium phosphate 4. Microcrystalline cellulose 634. For water sensitive products, which of the following diluents should be used in formulations? Reference Guide for Foreign Pharmacy Licensing Exam ‘Questions and Answers- 2nd Revised Edition 1 Lactulose W.—_Diabasic calcium phosphate IL Calcium sulfate Tonly Land Ionly Mand IIT only 1, I and II only pegs 635. The reactionthat may be caused by certain amine drugs in the presence of lactose diluent is known as : a. Redox reaction b. Leonard reaction c. Millard reaction d Axis reaction 636. Which of the following is an ex- ample of hydrolysed starch ? a Emdex db Cerelose c. Dextrose d— Avicel 637. Which of the following diluents is used for manufacturing chewable tablets ? a Lactose b. Menthol Sorbitol @.—— Microcrystalline 638. Which of the following is/are naturally occurring gums? 1 Acasia IL Tragacanth ML Gelatin a Tonly b. Land I only c. TandIIlfonly dL, Mand Mfonly www.pharmacyexam.com Krisman 639. Which ofthe following ingredients is employed as a disintegrantin the manufacturing of tablets ? Sorbitol Dibasic calcium phosphate AcDi-sol 50% solution of glucose eege 640. Which of the following is true about the coating of tablets ? 1. Itmasks the taste, odor or color of the drug I It controls the release of drug from the tablet. IIL. _Itprotects the drug from disaster effect of stomach acid. Tonly Tand Il only Tl and II only 1, Hand IIT only peep 641. Lubricants are intended to: a. Increase the flow property of gran- ules. b. Reduce the friction between the tablet and the walls of die cavity in which the tabletis generally formed. ¢. _ Disintegrate particles of the tablet in the stomach. 4. Improve the color property of drugs. 642. Inwhich of the following drug informa- tion literature are the citations arranged alphabeti- cally by first author and then by subject head- ings? 9 Reference Guide for Foreign Pharmacy Licensing Exam ‘Questions and Answers- 2nd Revised a. Pharmacy Law Digest b. Clinical Pharmacy and Therapeutics a Index Medicus d. PDR 643. Which ofthe following seferences is use- falto find out which drugs have been withdrawn from the U.S. market ? a. AMA Drug Evaluation b. — Drugdex ce D-List ad PDR 644, Which of the following reference sources helps for physical identification of tablets and capsules ? t Identidex I Red Book TI Blue Book a Tonly b. Land Honly cc. Tand Ill only a all 645, Tofind outthe wholesale and retail price of particular product, one can use which of the following 1 TheAmerican Druggist Blue Book Drug Topics Red Book Ml, — DeHaen Drug Data a Tonly b.— TandIonly c.— Tand IT only d. All 646. Which of the following HMO models has the highest percentage of HMO membership ? a. Staffmodel HMO_ www.pharmacyexam.com Krisman b. Group model HMO c. Network model HMO 4. IPA model HMO. 647. Physicians participating in IPA HMO’ are normally reimbursed by L Capitation I Discounted FES IL Salary a. Tonly b. Land Ionly c. Hand only ad All 648. 1 millimicron is equal to how many angstroms ? eo 1A b. 100A" ic 1005 4. 1000.A° 649. Which of the following is TRUE abou a USP standard dropper? a ‘The external diameter of the dropper must be 3 mm atits delivery end. I Whenheld vertically itshould deliver 20 drops of water, the total weight of, which is between 0.9 gand 1.1 g at 25°C. TI Itwas first recommended by the Brussels Conference of 1902 for international adoption. a Tonly b, Land Jonly’ cand Ill only a All Reference Guide for Foreign Pharmacy Licensing Exam Questions and Answers- 2nd Revised Edition 650. Whatis the specific gravity of 2L of Car- bon tetrachloride having a weight of 2500 gm ? a 225) 08 ce 0.008 a 1.25 651. Which of the following isthe correct unit for density ? a govec b.—celgm cc. gimme 4d gmem/mm? 652. What would be the pH of the solution, if the concentration of hydrogen ions in the 0.0001 g-ion/L.? a3 b. 1 e 2 da 4 653, Ina statistic, Bias refers to a. the closeness of measurement. thereproducibility of result, systematic difference from the true value. d. All 654. Vasculitis angitisis defined as: a. Asenseof suffocating. b. A patchy inflammation of the walls of small blood vessels. ©. Anexcessive variation in size between individual red blood cells. d. Failure ofthe kidney to produce urine. 655. Which of the following abbreviations is used to describe a Podiatrist ? www.pharmacyexam.com 2 0.09M solution of ammonium chloride. Krisman a MD b= DDS ©. DPM id NP 656. How much sodium bicarbonate is re- quired to prepare 300 cc of 0.05 N solution of NaHCO, ? [ molecular weight = 84 gm/mole] 8.4gm 1.26 gm 4.2gm 0.84 gm aor CH,=CH, is classified cA a. SP* hybridization b. — SP*hybridization c. SP hybridization da. P* hybridization 658, Which of the following bonds requires the greatest amount of energy ? apge 659. If i partof solute requires 10 to 30 parts of solvent, it should be defined as: Soluble Freely soluble Slightly soluble Sparingly soluble eoge 660. Calculate the [H30°] concentration of a \ssume that Kb for ammonia is 1 x 104 and Kwis 1 x 10 a 102 b. -24x105 81 Reference Guide for Foreign Pharmacy Licensing Exam Questions and Answers- 2nd Revised Edition cox 10e da 9x10" 661. Which of the following class of surface activeagentsis widely usedas apreservativerather than surfactant? I. Cationic agents —Anionicagents UI Amphoteric agents a Tonly b. Land I only Wand IIT only 4 All 662. The small intestine is the primary site for absorption of an orally administered drugs be- cause I ‘The intestine epithelium is more permiable to drugs than the stomach, I The absorptive area of the intestine is greater than that ofthe stomach Hi Thetotal blood flow through the intestine capillaries is much greater than the stomach. a Tonly b. Land only c. MandIltonly a All 663. Which of the following is NOT TRUE. about enzyme inhibition reactions ? a. Theyoccur much more slowly than induction reactions. b. _Theyare the mostclinically significant type of interactions. www.pharmacyexam.com Krisman c.Theinhibition of substrate metabolism tends to begin as soon as sufficient concentrations of the inhibitors are reached. 4. Elderly patients are at greater risk from enzyme inhibition reactions. 664. Theelderly are more susceptible to which of the following conditions due to the decline in baroreceptor function ? 2 Hypertension b Tachycardia c ——_Bradycardia d-—— Orthostatic hypotension 665, The most widely prescribed ERT for treating menopause symptoms is a. Estropipate b. — Conjugatedestrogen 1¢/972" 4 cc. Estradiol 4. ‘Medroxyprogesterone 666. As with most menopause symptoms, ‘hot flashes are due to a. Estrogen deficiency b. —_Estrogenexcess c. Progesterone deficiency Progesterone excess 667. The symptoms of BPH are caused by a, pressure exerted by the prostate gland on urethra b. relaxation of bladder muscles ¢. shrinkage of the prostate gland a relaxation of the prostate gland muscles 668. The primary cell type responsible for the Jong-term effects of asthma is 82 Reference Guide for Foreign Pharmacy Licensing Exam Questions and Answers- 2nd Revised Edition Soe Basophil T lymphocyte B lymphocyte pegs 669. The major immunoglobulin associated with asthma is a IgA b IgM c eB a IgG 670. Which of the following factors triggers an asthma attack ? I Tobacco smoke TI Exercise MM — Coldair a lonly b. Land Ionly «Mand IIT only a All 671, Which of the following is the clinical hallmark of asthma ? Sneezing Coughing Wheezing Dozing pegs 672. Wheezing is defined as: a. _apersistent cough or tightness in the chest. b. _ahigh-pitched sound caused by turbulent airflow passing through an obstructed airway. ¢. _adegranuation of eosinophils and release of chemical mediators. 4. abronchoconstriction, swelling, and an excessive mucous secretion in the airway. www.pharmacyexam.com Krisman 673. Blood dyscrasias that affect all three blood cell lines are defined as: a. Thrombocytopenia b. —Agranulocytosis ©, Leukemia d.—Pancytopenia 674. Which of the following drug-induced hematological disorders are associated with the highest morbidity and mortality rates ? Hemolytic anemia a b.— Leukemi c. Aplastic anemia d.—_Agranulocytosis 675. Which of the following drugs causes hemolytic anemia in patients with G6PD de- ficiency? 1 Cortrimoxazole 1 Dapsone TM Primaquine a Tonly b [and ITonly ¢ Hand Ilfonly ds All 676. Which of the following is a sign and symptom associated with drug-induced aplastic anemia? 1 nL Ill Frank bleeding a Tonly b. Land Ilonly cc. Hand Illonly 4 All 83 Reference Guide for Foreign Pharmacy Licensing Exam Questions and Answers- 2nd Revised Edition 677. A patient’s blood report indicates granu- locytes count of 1000 cells/mm, This condition isdefined as: 1 Granulocytopenia I Leukopenia I Agranulocytosis Tonly Land Il only Mand II only All pegs 678. The term that normally describes for a // Sug that when combined with a protein ren- ders it antigenic is a, Immunophilic b. — Heptane cc. Antiglobin d.——Thromboxatic 679. Which of the following is the most com- mon causative agent for Community Acquired Pneumonia ? a. S.pneumonia b.- Mpneumonia cc. — Saureus d.— Cypneumonia 680. Which of the following is NOT TRUE, about Xerostomia ? a. Tteauses dental caries. b. —_Ttcauses difficulty in swallowing medications. €. It causes significant distress to patients. 4. It contributes excessive salivation. 681. Theelderly are more susceptible to which of the following conditions due to a decline in baroreceptor function? www.pharmacyexam.com Krisman a Hypertension b Tachycardia c — Bradycardia d—_ Orthostatic hypotension 682. Temazepam and Lorazepam are pre- ferred benzodiazepines for use in the elderly because a They donot undergo phase I reactions. b They do not undergo phase II reactions. ¢ They extensively excrete by first pass metabolisms. They have limited first pass metaboiism, 683. Amlodipine is classified as: Beta blocker ACE inhibitor Ca-channel blocker ‘Vasodilator sege 684. An amine base that is not metabolized and has a PKa of 8 will be reabsorbed from the renal tubules most quickly if the pH of the urine is adjusted to: 5 6 3 i Boge 0 685. Ten hours after 750 mg of a drug is ad- ministered by IV injection, a patient’s plasma con- centration is 20 meg/ml. If the half-life of this drug is S hours and the minimal effective concentration (MEC) is 3 meg/ml, how many hours after the first dose should a second dose be administered? a. 10hours b. 22 hours c. 37.Shours d. | 1Shours Reference Guide for Foreign Pharmacy Licensing Exam ‘Questions and Answers- 2nd Revised Edition 686. Thefollowingtwosystems were prepared without surfactant. System A System B Volume of oil 10mi 100mi Droplet of radius 1 mem mem Interfacial tension _80dyne/em 8Odyne/em Volume of water 250 ml 250 ml Which of the following statements about the thermodynamic stability of SystemA is true? a, Itis as stable as system B b. Its 10 times more stable than system B. c. It is twice as stable than system B. d. tis 50 times more stable than system B. 687. The radioactive decay of Mobidium has arate constant of 5x 10+ year. How many years will it take for 90% of the Mobidium initially presentto degrade ? (The amount initially present can be specified as 100%) a 25 b. 2345 ©. 3243 4. 4606 2 688. A solutioncontains methyl acetate (0.1M) 7 and sodium hydroxide (0.1M). If the rate con- stant for this reaction at 25°C is 1.082 liters (mole.min), how many minutes will it take for the concentration of methyl acetate to fall to 0.09 M? a 1.026 b 23.45 © iis a 543 689. Thehalf-life of a drug is 12days.A single / 300 mg dose of the drug yields an [AUC] value www.pharmacyexam.com Krisman (of 360 megfn/nt. In micrograms per mililiter, what plasma level will result ata steady state if this product is given twice a day and is 80% bioavailable? a 10 bd 45 ec 30 de e 90 ‘24-hour urine sample is collected from ~S apatient who has noninsulin diabetes mellitus and stable creatinine level of 2 mg/dL. The sample shows atotal volume of 1400 ml and creatinine concentration of 100mg/dL.. In milli minute, whatis the approximate glomerul tion rate for this patient ? iva 50 orf Wie b 30 . 75 a 120 691. Which of the following NSAIDS is indi- cated forthe treatment of dvctus arte ‘maitare infants? a, Tbuprofen b. —_Indomethacine c.—— Rofecoxib &—— Nabumetone 692. Which of the following enzymesisa rate limiting step in glycogenolysis? a, Glucose-6-phosphate b. _ CAMP-dependent proteinkinase c. Glycogen synthase d. Glycogen phosphorylase 693. Whichof the following is the most potent ‘estrogen in the human body ? Reference Guide for Foreign Pharmacy Licensing Exam Questions and Answers- 2nd Revised Edition Estradiol Estrone Estriol oe Tonly Land Il only Mand III only Al aece |. Which of the following is NOT TRUE ‘about Neonatal tetany? 1 Ttisagenetically fatal disorder. I Ttoccurs in newborns of mothers with hyperparathyroidism. II It disappears as soon as the infant’s parathyroid gland responds normally. a Tonly b — Tand Ionly © Hand {llonly dal 695. Which of the following is useful in treatment of inappropriate secretion of ADH? a. Doxyeyeline b. Demeclocycline e Chlorpromazine d. Rivastigmine 696. Which of the following has the longest hali- life? a. Ciprofloxacin b.— Clofaimine cc. Sulfadiazine d.——Dicloxacilin 697. The partial supply of Controlled I drugs. should be filled within a. 12hours b. 72 hours www.pharmacyexam.com Krisman c. — 48hours d.— 4hours 698. Which of the following drugs may be found under the Schedule II classification? 1 Fentanyl TL Amphetamine TI Methylphenidate a Tonly b Land Wonly ¢ Mand Iionly dll 699. The DEA requires that the inventory of Schedule II should be done: every week every six months every two years every five years aege 700. The partial dispensing of Schedule IIT controlled drugs should be done: a. everyyear b. 6 months cc. — everyweek 4. 3months 701. Which ofthe following auxiliary labelsis/ are required before dispensing Biaxin (Clarithromycin) suspension ? 1 Take with food, Donotdiscontinue. TM Store in refrigerator. Tonly Tand only Mand III only All aoge 86 Reference Guide for Foreign Pharmacy Licensing Exam Questions and Answers- 2nd Revised Edition 702. A patientis taking Precose (Acarbose), 100 mg three times a day. The pharmacist has to advise the patient to regularly check his a AST b scr c INR der 703. An example of HMG-COA reductase enzyme inhibitoris/are Simvastatin TL. Pravastatin T. — Lovastatin a Tonly b.— Tand Ifonly ¢— TMand If only a all 704, Which of the following drug(s) is/are used for obsessive compulsive disorder? 1. Clomipramine |. Imipramine TM Amoxapine only Land If only Mand Iifonly All pore 705. A patient is using an inhaler for the first time. As a pharmacist, you would advise himto 1. Hold the Aerochamber and shake vigorously 304 times. I __Breathein slowly and deeply through the mouth until you have taken a full breath, I. Holdbreath for 5 to 10 seconds and repeat steps. www.pharmacyexam.com Krisman a Tonly b. Land Ionly c, Hand Ifonly a All 706. KetoroiacTromethamine can be. given L Orally one 1 LM. mL. a Tonly b. Land only c. Hand Ill only d. All 707. Which of the following iscommonly used asa household oxidizing agent? Formaldehyde Benzalkonium chloride Hydrogen peroxide Bihylene oxide BoeoR 708. The primary mechanism of action of Esomeprazole is aH, receptorantagonism b —_5-HT,receptorantagonism © Dy receptor antagonism dG. proton pump inhibition 709. Erythromycin is a choice of drug for treatment of a. Onychomycosis b. Legionnaires’ Disease c. Herpes simplex a HIV 710. Which ofthe following statement(s) about ‘Talwin NX is/are true? 1 ‘The active ingredients of Talwin NX are Pentazocine and Naloxone HCI. 87 Reference Guide for Foreign Pharmacy Licensing Exam Krisman Questions and Answers- 2nd Revised Edition Il. Itisintended for LV. and. Muse. 714, Microorganisms thatestablish permanent residence without producing disease are known Ill, Naloxone HCI (0.5 mg) has profound as antagonist activity when given orally. a. Normal flora a Tonly b. —Transientflora b. — Tand Monly c. Opportunistic pathogen Hand Hl only 4. Microbial antagonism a Al 715. Allofthe following drugs are serotonin TIL. All of the following drugs may raise ake inhibitors used in the treatment of de- the blood serum concentration of Theophy|- line EXCEPT rey pression EXCEPT a ‘Venlafaxine a. Allopurinol b. Paroxetine b. — Primidone _-Fivoxetins ©. Cimetidine a Tran d. Ciprofloxacin meypromine 716. Otitis medias generally caused by 712. Which of the following drug(s) is/are . a ema! indicated for treatment of chemotherapy in- L Hinfluenze duced nausea and vomiting? I. S.Pneumonia {1 M.Pneumonia 1. Ondansetron I Granisetron in) TI. Dolasetron b. Land Ionly c. Hand Ill only a. Tonly 4. All b. Land ITonly c. Hand 1fonly Ai ‘Al ¥ 717. Otitis externais generally caused by: 4 713. Mr. Keit istaking Diphenydramine fg exidermis elixirto treathis allergy problem. Suddenly one . Pacruginosa nighthe suffers from flushing, throbbing headaches, a Spyrogen breathing difficulty, nausea, vomiting, weakness, and blurred vision, These symptomsare because of 718. Mrs. Rey wants touse an Estraderm sys- tem for the first time. She comes to the pharma- cist and asks how to use the transdermal system. oe ‘The pharmacist may counsel her ofall ofthe fol- pe eiticad lowing EXCEPT cc. Chlorpropamide 8 4. Diphenhydramine a. Thepatch should be placed on aclean, nonhairy and dry area of skin www.pharmacyexam.com 88 Reference Guide for Foreign Pharmacy Licensing Exam ‘Questions and Answers- 2nd Revised Edition b. Thesite of the application must be rotated at least every week. The breastis the best site for the application of the patch. @—_Thesystem should be applied immediately after opening the patch. 719. Which of following about Didanosine is/ aretrue? 1. Tthas a more prolonged duration of action than Retrovis, with least bone marrow suppression property. I The percentages of patient survival with Didanosine is higher than with Retrovir. I. Antacids may reduce or prolong the absorption of Didanosine, so an interval of atleast 2hours berween administration of Didanosine and antacids is required. a Tonly b. — TandIonly ¢. IHandIIlfonly a All 720. Cold sores or fever blisters are normally caused by: a HSV-1 db. -HSV-2 c. Varicella zoster ¢.—Variola minor 721, Which of the following antibiotics should be avoided by patients who are suffering from GOPD deficiency? a Tetracycline b. Penicillin ‘www.pharmacyexam.com eee eee cect Krisman. ©. Sulfonamide d.— Cephalosporin 722. Trachomaisnormally caused by: a. Tineacapitis b. —Sporotrichosis c. Corynebacterium diptheria 6. Chlamydia trachomatis 723. Which of the following drugs should ‘de used for vitamin K deficiency? a. Ergocalciferol b. —_Phytonadione Ascorbic acid 4. Pyridoxine 724. Mr.Xissuffering from hypersensitivity reactions with Pentazocine. Which of the follow ing drug(s) should be avoided by Mr. X because ofhisallergy? 1 Talacen 1. TalwinCompound Mm. TalwinNX Tonly Land Il only Tiand Wonly All sess 725. Mr. Keit stnsto take a Pancrelipase cap- sule for treatment of nutritional disorder. After ‘one week, he lost control of his blood glucose level. When he went tothe pharmacy and asked the pharmacist about his problem, the pharmacist ‘counseled him that 1. Panerelipase enzyme increases the conversion of the complex carbohydrate toglucose. 89 Reference Guide for Foreign Pharmacy Licensing Exam ‘Questions and Answers- 2nd Revised Edition Il. Panerelipase enzyme generally triggers the blood glucose level by inhibiting insulin secretion. TI. Pancrelipase enzyme increases insulin secretion. a. Tonly b. Land IT only c. Hand IIT only all 726. Which of the following medication(s) should be avoided with Ketorolac? LPlicamycin I. Valproic acid TL Moxalactam a Tonly b. Land only ¢. Hand If only a All 7271. Whichof the tollowing drug(s)is/areAn- giotensin II receptorAntagonist(s) ? 1 Valsartan Losartan I Irbesartan a Tonly b Land ITonly c Mandillonly dll 728. Which ofthe following informations sare true about Infectious Mononucleosis? 1. Ttiscaused by Epstein Barr virus. I tis characterized by enlarged and tender lymph nodes, an enlarged spleen, fever, sore throat and headache. www.pharmacyexam.com BE pege 729. Krisman Itis commonly spread by sneezing. Tonly Tand Tonly Mand Iltonly All Which of the following TCA is used in the treatment of nocturnal enuresis? aege 730. Doxepin Imipramine Protriptyline Amoxapine Corlopam (Feoldopam) is indicated for short-term (up to 48 hours) management of se- vere hypertension, Itis classified a(n) a Alpha-I receptors agonist b —_Beta-2 receptor agonist ¢ — Dopamine-D1 receptor agonist ¢ Histamine-H1 receptoragonist 731. Deferoxamine mesylate is administered by 1 LM. route — LV.soute MI. S.C. route a. Tonly b. Land Ionly c. Mand Ilfonly a All 732. Metoclopramide produces its antiemetic effect by 1. Inhibiting stimulation ofthe chemoreceptor trigger zone. IL UL Increasing GI. motility. Increasing the rate of GI. emptying. 90 Reference Guide for Foreign Pharmacy Licensing Exam Questions and Answers- 2nd Revised Edition a Tonly b — TandIfonly c. Tand Ill only dll 733. Burkitt's lymphomas caused by: S.aureus Tpallidum Epstein Barr virus Toxoplasma gondii Bese 734, _Allofthe following drugs are useful in the treatment of status epileptics EXCEPT 50% dextrase solution Diazepam Pherytoin Triazolam ane 735. Which of the following about Trimethobenzamide is true? 7 Itis indicated for control of nausea and vomiting. I. Itisavaitable in caplet, injection and suppository form. II. NoAB or AT rating is listed for ‘Trimethobenzamide. Tonly Land Ilonly Mand II only All 736. Which of the following is NOT TRUE, about Vibramycin? I. _Itisindicated for the treatment of SIADH. IL Itcanbe safely used in renally impaired patients. www.pharmacyexam.com, Krisman TI. Itmay be taken with food or milk to reduce GI. invitation. Tonly Tand Il only Mand Ilonly All pegs 737. Sunscreen products should cover a. 500nmUVIight b. 400 nm UV light ¢. 320nmUV light 4. 100mm UV light 738. Which of the following is known as the pacemaker of the heart? a SAnode b. — AVnode ©. Purkinje fibers 4. Tricupsid valve 739, Which of the following is/are true about Zofran? I. _Itisindicated for the prevention of cancer chemotherapy induced nausea and vomiting Il It is available in tablet, injection and oral solution forms. II, Theactive ingredient of Zofran is Ondansetron HCI. Tonly Tand Ionly Mand IM only All eege 740. Which of the following hormones is se- creted by the posterior pituitary gland? o1 Reference Guide for Foreign Pharmacy Licensing Exam Questions and Answers- 2nd Revised Edition a. Prolactin b. Follicle stimulating hormone cc. Vasopressin 4. Lateinizinghormone 741. Which of the major serum proteins is in- volved in the protein binding of drugs ? a. Allpha-1 acid glycoprotein b. Lipoprotein Albumin a Beta-1 acid glycoprotein 742. Which of the following is/are TRUE, about Advair Discus ? 1 Itisacombination product of Fluticasone and Salmeterol. a ‘The normal therapeutic recommended dose of Advair is twice a day. I _Itshould be immediately used for prevention of acute attacks of asthma, a. Tonly b. — TandIfonly cc. Mand lfonly a All 743. Allofthe following drugs are vseful dietary supplement products of pancrelipase enzymes EXCEPT a Cotazym, b. — Pancrease. © Zymase. d.— Pulmozyme. 744, Fameyclovir is generally indicated forthe treatment of aS. Aureus b.— C. Albican www.pharmacyexam.com Krisman cc. MiLeprae 4. H.Zoster 745. Whichof the following is/are true about Pentam-300? L Itis mainly indicated for treatment of P. Carini Pneumonia. TI, Severe hypotension will be observed when given by LM.or LV. TIL Prescriptions for Pentam-300 can be filled by NebuPent. a Tonly b. Land Ionly e. THand IMlonly a All 746. Mesalamine generally acts by a. Binding to 30_ ribosomes. b. _ Binding to 508 ribosomes. ¢. Local bactericidal effects oncolon bacteria, 4d. Inhibiting prostaglandin synthesis. 747. The normal range of prothrombin time is a 1 to 5 seconds b, 12 to 15 seconds ¢. 2010 30 seconds d. 30 t0 60 seconds 748. The body mass indexis calculated by: a. Weight (kg) /body surface area b. Weight (Ib)/height (inches? c. Weight (kg) / height (meter)? 4 Weight Ib) /height (meter) 749. Renal clearance of phenobarbital can be increased by administering 92 Reference Guide for Foreign Pharmacy Licen: Krisman ‘Questions and Answers- 2nd Revised Edition a. Sodiumbicarbonate a. Tonly b. —Sodiumcitrate b. Land only c. Magnesiam citrate Mand If only d. Potassium chloride a all 750. Rifabutin doses should be reduced by 75% from the recommended dose when 754, Allofthe following drugs are indicated in the treatment of organ transplantation as immu- coadministered with nosuppressive agents EXCEPT a Lopinavir a. Azathioprine b.— Gemfibrozit b. Cyclosporine < — Glipizide Tacrolimus d— Olanazapine 4. Rocuronium 751. Severe headaches are commonly asso- 755, Which of the following is/are first-line ciated with antihypertensive agent(s)? a Isosorbide dinitrate L HCTZ b. Furosemide I Atenolol ¢ — Bumetanide UL Captopril 4. Propranolol a Tonly 752. Posicor (Mibefradil) is indicated for the b. Tand Honly treatmentof c. [and IIlonly a all a CHF : b. Hypertension 756. The probable mechanism of action of ©. Arthritis Losartan is da BPH 753. Which of the following is/are true about By inhibiting the enzyme which is responsible for conversion of Ace Tin Accolate? Ace Il. b. By vasodilation of peripheral arteries, 1. Theactive ingredient of Accolate is ¢. ByblockingAlpha-1 receptors. Zafirlukast. 4. Byblocking AT type II receptors. Tl —_Itacts by antagonizing LTRA- receptors which are responsible for the 757. Skin protection factor (SPF) must be release of SRSA. 1. Sormore, but less than 14 M10 or more, but not less than 14 M15 ormore MW. _Itisnota bronchodilator and should not be used for treatment of acute a. Tonly attacks of asthma, b. Land I only ¢. Mand Ilfonly a All www.pharmacyexam.com 93 Reference Guide for Foreign Pharmacy Licensing Exam Questions and Answers- 2nd Revised Edition 758. Capsofungin is indicated to treat infec tions caused by a. Herpes virus b. ——Sipneumonia ce A.fungi d—V.cholera 759. The chance of hypoglycemia is the least when therapy is maintained with 1. Acarbose 1. Metformin I. Troglitazone only Land only Mand Ill only All aese 760. Which of the following is/are bulk faxatives ? 1. Metamucil T. — Citrucel TL. — Miolan a Tonly b. and ITonly c. Tand Ilfonly a al 761. The primary mechanism of action of Pan- toprazole isa a. H,receptorantagonism. b. S-HT,receptorantagonism ©. D, receptor antagonism 4. Gl. proton pump inhibition 762. Which of the following drug(s) should be avoided during Rezulin therapy? www.pharmacyexam.com ve BRO Krisman Cholestyramine Colestipol Terfenadine Tonly Land Honly Mand Iitonly All 763. All of the following mentioned drugs produce cardiac toxicity (like cardiac arrhyth- ‘mia) when givenin combination with Ketoconazole EXCEPT a. Astemizole b. —Terfenadine cc. — Loratadine 4. Cyclobenzaprine 764. Which of the following is/are true about NSAIDs? L IL MU. eese NSAIDs are indicated for treatment of analgesia, dysmenorrhea and bursitis. GL ulceration and bleeding have been reported with the use of NSAIDs. Any OTC NSAID should not be used ‘more than 15 days for pain or 10 days for fever. Tonly Land only Mand Itonly All 765. Which of the following about Lovenox is/are true? 1 Theactive ingredient is Enoxaprine Na. I. _Itisalow moiecular weight heparin 94 Tris available ina 30 mg/0.3 ml injection, Yonly Tand Il only Mand ITonly All 166. ‘The major side effect of Cylert (Pemo- ine) that restricted its use in individuals is a. Renal failure b. Heart failure c.—_ Liver failure d@ Brain failure 767. The probable mechanism of action of Ga- Baril (Tiagabine) is a Alpha-t receptors blockage b. Enhanced activity of GABA c. _ Beta-I receptor agonists 4 Serotonin SHT, receptor blockage 768. Theactive ingredient of Combiventis/are 1 Ipratropium Br IL Albuterol $04 WL. Metaproterenol a Tonly b. Land Il only ¢. MandIHlonly a all 769. Which of the following drug(s) is/areAn- giotensin type I receptor antagonist(s)? T Losartan Tr Irbesartan + HCTZ TM Zileuton a. Tonly b. Land I only www.pharmacyexam.com snce Guide for Foreign Pharmacy Licensing Exam ms and Answers- 2nd Revised Edition Krisman c. HandiIlonly a All 770. Which of the Insulins has the shortest du- ration of action and the fastest onset of action ? a Humulin-R be. Lantus ce. Romalog d. Novolog 771. ‘The strength of folic acid in OTC vita- ins should not exceed ay omg b 10 mg © 04mg & 10 meg 772. Before initiating therapy with Paclitaxel, iisrecommended that the patient be pretreated with 1. Corticosteroid I. Diphenbydramine HI, H, Antagonist a Tonly b.— TandIfonly c. Mand Mf only d. All 773. Which of the following drugs is contrain- dicated in the treatment of B.PH.? 1 Diphenhydramine Ti, —_Phenylephrine/PPA/Gauifenesin TM —Atropin only Land Ionly Mand Honly All eese 95 Reference Guide for Foreign Pharmacy Licensing Exam Krisman Questions and Answers- 2nd Revised Edition 774. Sumatriptan is mainly indicated for the treatment of a. Cancer therapy induced nausea and vomiting, b. Migraine c. Depression d. Seizure 775. Methicillin resistant infection would be treated by a b. Cephalexin c. Vancomycin d Ceftriaxone 716. Which of the following antihypertensive agents helps patients on diabetic treatments? a. Verapamil b. Enalapril c. _ Hydrochlorothiazide dd Amiloride 777. Theactive ingredient of Compound W is a. Sulfur b. Aspirin c. — Salicylicacid 4. Coal tar 778. A patient is on Antabuse for his treat- ment of alcohol withdrawal. Which of the following product(s) should not be taken with Antabuse ? 1. Theophyliin Na glycinate elixir I. Theophyllinelixir ‘Theophyllintablet a. Tonly b. — TandIfonly ¢.— TMand If only a All ‘www.pharmacyexam.com 779. A depressed person with anxiety disor- der should be treated with a. Doxepin b. Nortriptyline c. Fluoxetine Amitriptyline 780. The mechanism action of Fluoxetine HCL reuptake inhibitor of norepinephrine. reuptake inhibitor of epinephrine, reuptake inhibitor of serotonin. reuptake inhibitor of dopamine. aege 781. Ondansetron is mainly indicated for the: a. Prevention of hypertension, b. Prevention of nausea and vomiting induced by cancer chemotherapy. cc. Prevention of migraine attack. 4. Preventionof angina, 782. All of the following drugs should be avoided with Ketorolac EXCEPT a. Meclizine b. — Cefotetan c. Goldcompound d. Methotrexate 783. Which of the following hormone is se- creted by the kidney in response to a reduction in the amount of oxygen that reaches tissues? a. Calcitonin b. Erythropoietin ce. Insulin d. Melatonin 784. Carbamazepine is mainly indicated for 96 Reference Guide for Foreign Pharmacy Licensing Exam Questions and Answers- 2nd Revised Edition a Trigeminal neuralgia b Ambythmia © Hypertension Depression 785. Erythromycinis the preferred agent for treatment of Legionnaire's disease Mycoplasma Pneumonia Campylobacterial infection Bae only Land IT only Hand 1 only All aoge 786. All of the following drugs act 2s anti- hyperlipidemic agents by inhibiting HMG- COA reductase EXCEPT a Lovastatin b. Simvastatin cc. Fluvastatin d.—— Clofibrate 787. After initiation of therapy, Mr. Teddy sud- denly suffers from spasms of the face and neck. Which ofthe following drug(s) should be given to alleviate the above symptoms? I. Diazepaminjection I. Diphenhydramine 1. —_Benztropine Tonly Land Il only Hand Illonly al ae ge 788. Cholestatic hepatitis is commonly asso- ciated with a —_Erythromyein www.pharmacyexam.com Krisman . _ Exythromycinethylsuccinate c. Erythromycin stearate 4, —_Erythromycinestolate 789. Which of the following prostaglandins is ‘a potent mediator of asthma attacks? a. PGD2 b. PGE2 c. PGF2 ad PGIZ 790. The purity of Insulin can be measured Content of Proinsulin Content of Acid buffer Separation of Dextroinsulin Degradation of Insulin at elevated temperatures 791. Mrs, Xisa pregnant woman, Which of the following antithyroid agents should be sug- gested by her pharmacist? L PIU 1. —PIMASyrup MI, — Methimazole a. Tonly b. — TandIonly ¢. Hand {lfonly d. All 792. Which of the following drugs is/are dopamine receptor agonists ? 1 Bromocriptine UW —Pergolide I. Tranyleypromine a Tonly b. — TandIfonly cc. Hand tonly d. All 97 Reference Guide for Foreign Pharmacy Licensing Exam Questions and Answers- 2nd Revised Edition 793. The patients should avoid sunlight when he or she ison Hydrochlorothiazide Thioridazine Tetracyclin HAT a Tonly b. Land Ionly cc. Hand Ilfonly a all 794, Which of the following is NOT TRUE, abouthistamine? a. Itincreases the contraction of smooth muscles, b. —_Itdecreases force of contraction and rate of heart. ¢. __Itstimulates gastric acid secretion. 4. Itisreleased in large amounts after skin damage. 795. ‘The normal therapeutic blood serum con- centration of Phenytoin is 101020 mg\ml 101020 meg\ml 301050 meg\m! 101020 meg\m! esse 796. Allof the following are adverse effects of gold compoundsEXCEPT a Diarrhea b. Abdominal pain Stomatitis 4. Lipowtrophy 797. Which of the following antihistamine(s)is! are nonsedative antihistamines ? i Cetirizine Hl Terfenadine I. — Astemizole www:pharmacyexam.com Krisman a Tonly b. — TandIlonly c. Hand Ifonly da All 798. The minimum weighable quantity for class-A prescription balance is a 100mg b. 80mg ce. 60mg ad -120mg 799. Which of the following is/are true about Bupropion? I. _Itis presumed to act on dopaminergic and noradrenergic pathways involved innicotine addiction and withdrawal. IL Ithelps reduce the urge to smoke. IIL. _Ithelps reduce nicotine withdrawal symptoms a Tonly b. — TandIonly c. Hand MTonly da All 800. Troglitazone is mainly indicated for reat- mentof a. Hypertension b. Diabetes cc. Hyperthyroidism a CHE 801. Mr.Zecis taking Verapamil SR 240mg for treatment of his blood pressure. The pharma- cist may tell him that he must be careful using the following drugsEXCEPT a. Atenolol yl Reference Guide for Foreign Pharmacy Licensing Exam Questions and Answers- 2nd Revised Edition b. Digoxin c. _Disopyramide d. Indomethacin 802. Which of the following drugs should not be used for more than 10 days for allergic con- Jjunctivitis? a. Ketorolac b. Lotwprednol c Azelastine 4 Tetrahydrazoline 803. The deficiency of which of the following, may increase the chances of bleeding? Red blood cells White blood cells, ‘lymphocytes “Thrombocytes peste 804. Which of the following drug(s) isare re- sponsible for pulmonary dysfunction? Bleomycin I. Bromocriptine I. Pentamidine Tonly Tand Ilonly Mand Hf only All Boge 805. Whichof the following Ca-channel block- ers is useful in the treatment of cerebral spasm? a. Amlodipine b. Nifedipine .—Nimodipine 4. Isradipine 806. ‘Trimethobenzamide should be avoided in patients who are suffering from www.pharmacyexam.com Krisman a, Ulcers b. Reye's syndrome c. Hypertension d. Depression 807. Which of the following is/are Bile-acid binding resins ? Cholestyramine Colestipol Colesevelam a Tonly Land IT only Mand IIT only All aege 808. Which of the following may be damaged inMultple sclerosis? Nephron Myelin sheath Parietal cells Bowman capsule 809. All of the following is/are true about Fentanyl TOSEXCEPT a. Fentanyl TDS system isavailable in 50,75 and 100 micrograms of strength. b. __Itiscontraindicated to use for management of mild to intermittent pain. ¢. ASO micrograms perhour dose should be considered as a safe dose for initiation of therapy. 4. Fentanyl patches should not be administered to children under the age of 12 or adults under the age of 18 whose weight is less than 110 Ibs. 99 Reference Guide for Foreign Pharmacy Licensing Exam Questions and Answers- 2nd Revised Edition 810. Furosemide isavailable in L Tablet I —Oralsolution II. Injection a Tonly b Land I only © Hand If only a All 811. Which of the following NSAIDs is a pro- drug? a. Diclofenac b. Ibuprofen ©. Diflunisal 4. Indomethacin 812. Valproic acid mainly acts . _byiincreasing the concentration of GABA. b. by decreasing the concentration of GABA. ¢. by increasing sodium influx in the brain, d. by decreasing firing of chloride ions in the brain: 813. The Food and Drug Administration di- vided drugs according to their potential to cause birth defects in 5 different categories. Which of the following categories indicates the highest risk to the developing fetus? a. A & 8 ec a x 814. Which of the following sedative hypnotic agents is useful as an antiemetic agent for treat- ‘ment of cancer chemotherapy induced nausea and vomiting? www.pharmacyexam.com aege Krisman Oxazepam Clonazepam Prazepam Lorazepam 815. Which of the following glands secretes Melatonin? a. Pituitary b. Thyroid ce. Pineal d. Pancreas 816. A patients has “High Cholesterol” if his total serum cholesterol value is greater than eege 120 mg/dl 360 mg/dl 180 mg/dl 240 mg/dl 817. The primary mechanism of action of ‘Terazosin is a(n) a, b. ©. 4 beta-1 blocker alpha-1 blocker beta-2 blocker ACE inhibitor 818. A patients allergic toAmitriptyline. Which of the following seizure drug(s) should he avoid because of his allergy problem? 1 Carbamazepine I. Cyclobenzaprine UI. Valproic acid Na a Ionly b Land Honly ¢ Hand IMfonly ds Al 819. Ketonuriais normally found in patients withvafter 100 Reference Guide for Foreign Pharmacy Licensing Exam Questions and Answers- 2nd Revised Edition L__ Diabetes mellitus I. Severe vomiting WL. Starvation Tonly Land Ionly Mand IlLonly All pose 820. Which of the following is/are adverse ceffect(s) of Phenytoin ? 1. Gingival hyperplasia U. ——_Lupuserythematosus ML Ataxia a Tonly b. Land I only ¢. Hand ITonly 6. All 821. Hydrochlorothiazide causes all of the following EXCEPT Hypokalemia Hypercalcemia Hypouricemia Hyperglycemia Bose 822. Meniere's disease is characterized by: Episodes of deafness Vertigo Buzzing in the ears Ber Tonly Land only Mand IHonly All pose 823, A deficiency of which of the following may lead to goiter? a. Magnesium ‘wwi:pharmacyexam.com Krisman b. Aluminum c. — Todine d. Fluorine 824. Which of the following antifungal agent(s) {is/are useful in the treatment of oral candidiasis? I. Ttyaconazote I. Clotrimazole Ul, Nystatin Tonly Tand II only Hand Ill only All aece 825. Patients allergic to Cyclosporine need toavoid 1 Sandimmune HM — Gengraf ul Remeron a fonly b.— TandIfonly cand If only 4. All 826. Which of the following is NOT TRUE, about anorexia nervosa? a. Ttisagenetic disease characterized by severe weightloss. b. _Itiscommonly reported in adult females. cc. There is an excessive use of laxative or emetic agents. d. Fluoxetine is indicated for the treatment, 827. Zidovudine-induced anemiais best seated 101 Reference Guide for Foreign Pharmacy Licensing Exam Questions and Answers- 2nd Revised Edition 1 EpoteinAlfa T. Filgrastim TI Sargamostim a Tonly b. Land Tlonly cand If only a All 828. The overdose symptoms of Propoxyphene is/are I. __ Respiratory depression with cheyne stokes, 11, Hypoxia, pinpoint pupil construction and pulmonary edema. IIL Circulatory collapse. Tonly Tand II only Mand Il only all aoge 829. Mrs. X comes into the pharmacy to find the drug she used in Europe. Pharmacists may suggest al ofthe following reference materials EX- CEPT a. Matriandale Extra Pharmacopoeia b. Index Nominum ¢. United State Adopted Name 4. Dmug’s Fact and Comparison 830. Whichof the following about Methyldopa is/are true ? I Before the treatment is initiated. itis advisable to do a blood count and peri- odic blood counts should be done thereafter. I Hemolyticanemia occurs during therapy; remit promptly upon discontinuation of the drug. www.pharmacyexam.com ae ge Krisman Prolonged therapy with Methyldopa ‘may result in a positive Comb’s test. Tonly Tand only Mand HI only All 831. Hemoglobin tests measure | gm of He- moglobin per a. I ml volume of whole blood b. 1 dL volume of whole blood c. I gmvolume of whole blood d, liter volume of whole blood 832. All of the following statements about, Skelid (Tiludronate Na) are true EXCEPT Itsactive ingredients Tiludronate sodium, useful in the treatment of Paget’s disease of bone. Itis available in once a day dosing ‘The serum alkaline phosphate level ‘greatly reduces afterinitiation of therapy. Itcan be safely administered in renally impaired patients. 833. Allofthe following terms areelevated dur- ing acute attacks of asthma EXCEPT a. TLC b. FRV c. RV d. FEV-1 834. A female with her three-year old child ‘comes to the front counter of the pharmacy and asks for any nausea preventing medicine for her son. She says that her son has suffered from nausea and vomiting for the past three days. The phar- ‘macist may recommend: 102 Reference Guide for Foreign Pharmacy Licensing Exam Questions and Answers- 2nd Revised Edition a ‘Emetrol for controlling nausea and vomiting for three to four times day. b. —Toadmitherson immediately to the hospital. Plenty of fluid. 4. Overthe counter Meclizine. 835. A malignant disease in which the bone ‘marrow may produce excessive amounts of leu- kocytes is known as: a Polycythemia vera b. Thrombocytopenia ce. Anemia d. Leukemia 836. Which of the following is/are true about PEFR measurement? 1. The patients should measure and record PEFR twice daily for 2 weeks. U ‘The PEFR should be measured in the morning immediately upon rising and 10to 12 hours after the first PEF ‘measurement. II. The“personal best” PEFR is the highest PEFR found in a 2 week period. a Tonly b Land IT only © Tand IIT only dal 837. The onset of action of Insulin Lisprois a Sto 15min, b. 301060 min, ce. 810 1Ghours. dt 2 hours. www:pharmacyexam.com Krisman 838. Which of the following antihistamines is specifically used in the treatmentof vertigo? a. Loratadine b.— Clemastine ce. Cetirizine d.— Meclizine 839. The probable mechanism of action of Methotrexate is a(n) a. Policacid antagonist b. Prostaglandin Antagonist cc. Alpha-1 blocker d.—Cyclooxygenase enzyme inhibitor 840. Nightblindness is reported due to dietary deficiency of: a. VitaminC b.— VitaminA c. VitaminD a VitaminE, 841. The process by which the hypothalamus stimulates the pituitary to release particular hormones in response to a decrease of the hormone in blood is defined as: Positive Feed-Back Mechanism Negative Feed-Back Mechanism Releasing Factor Stimulation Factor Specific Hormone Stimulation aoge 842. Which of the following drugs is useful in the treatment of Cystic Fibrosis? a Ceredase b.— Pulmozyme c.— Protropine d.—Nutropine 843. Whichof the following is NOT a phar- macological action of Thyroid hormones? 103 Reference Guide for Foreign Pharmacy Licensing Exam Questions and Answers. 2nd Revised Edition a. __Itincreases oxygen uptake, BMR, and calorie production. b. __Itstimulates carbohydrate and protein metabolism. . __ Ttreduces serum concentrations of calcium by diposing blood calcium into bone. dTtincreases heart rate. 844, Which of the following drug(s) inhibit SRSA? 1 Zafirlukast I. Zileuton I. Cromolyn Na a. Tonly b. — TandITonly c. MandTIfonly a All 845, Which of the following benzodiaz- epines has a prolonged duration of action ? Alprazolam b.—Triazolam .— Flurazepam. 4 Oxazepam_ 846. Saquinavir should be stored in the re- igerator; however, once brought to room tem- perature itcan be used out within a day b. week c. 3months a Thour 847. Inmovable joints, which of the following serves.as a lubrication? www.pharmacyexam.com Krisman a Tendons b. Ligaments ¢. Synovial fluid d.— Microfilaments 848. The effectiveness of Warfarin therapy can be monitored by a. Bleedingtime b. Clotting time c. Prothrombin time d. Anticoagulant plasma concentration 849. Which of the following is NOT a phar- macological action of insulin? @. _Ttincreases utilization and oxidation of, sugar inthe tissues. b. _Itstimulates transports of glucose into cells, €. __Itstimutates breakdown of glycogen in muscles and the liver. d. _Itstimulates protein synthesis and growth. 850. Dryness of mouth, increased thirst, ir- regular heartbeat, muscle cramps or pain, nau- sea and weakness are signs of a. Digitalis toxicity b. —_Hydrochlorothiazide toxicity ©. Bisacodyltoxicity 4. Diazepamtoxicity 851. Allofthe follos g factors stimulate in- sulin secretion EXCEPT: a. Glucose b. Growth hormone Glucagon dQ. Starvation 104 Reference Guide for Foreign Pharmacy Licensing Exam Questions and Answers- 2nd Revised Edi 852. Digitalis glycoside increases: a. Influxofcalcium b. —_Influxof potassium Efflux ofsodium 4. Influx of sodium 853. Which of the following drug(s) is/are indicated for treatment of accidental overdose poisoning of drugs? Charcoal plus IL Actidose with sorbitol IM Actidose Aqua a Tonly b. Land Ionly ©. THandIMfonly a All 854, Calcium channel blockers are indicated for the treatment of angina because: 1. They decrease in myocardial contractility. TL They reduce oxygen consumption of the heart. II. They reduce coronary blood flow by constricting coronary blood vessels. a Tonly b. Land Ionly Hand IMfonly a All 855. Whichof he following islesslikely tore- absorb from renal tubules? a Glucose b. Water c.— VitaminA d.— Sodiumions www.pharmacyexam.com Krisman 856. A patienthas taken DiaBeta, (Glyburide) 2.5 mg for 6 months. He is recently diagnosed with hyperglycemia. Which ofthe following drugs is responsible for this elevated level of glucose ? a. Nifedipine extended release b. Omeprazole ¢. _Hydrochlorothiazide d.— Amitriptytine 857. Whichof the following is NOT asideef- fect of vasodilators? Hypotension Tachycardia Edema Weightloss eege 858. Which of the following drug(s) have been reported to interact with Cimetidine? Maalox Ketoconazole ‘Theophylline BRO Tonly Tand I only Mand If only Al Boge 859. Which of the followingca-channel block- ers has the least incidence of reflex stimulation of the sympathetic nervous system? 1 Amlodipine TL. Nifedipine WI Isradipine a Tonly b. Land Honly Mand {only d. All 105 Reference Guide for Foreign Pharmacy Licensing Exam Questions and Answers- 2nd Revised Edition 860. Whichofthe following is NOT presentin lymph fluid? a. Lymphocytes b. Urea c. Platelets, 4. Creatine 861. Hepatic toxicity is more common with 1. Iproniazid T. —Phenelzine M1, Tranyleypromine a Tonly b. — TandIlonly cc. and Ill only da All 862. The principal function of the lymph node isto: a, Providemutrientstocells. b. Remove waste material from cells. c. Protect cells against microbes and foreign particles. 4. Synthesize RBC. 863. Methylphenidate SR is generally indicated for treatmentof a. Parkinsons b. — Weightreduction c. Attention deficit syndrome 4. Painreliever 864. Which of the following drug(s) is/are carefully prescribed with Selegiline? 1 Meperidine TL Fluoxetine I. Tyramine www.pharmacyexam.com Krisman a Tonly b.— TandIonly c. Hand Ilfonly a All 865. Which ofthe following drugs was first ap- proved for treatment of HIV? Saquinavir Zalcitabine Didanosine Retrovir eese 866. Flushing isa major complication of Nia- cin therapy, and can be prevented by 1 Taking Niacin with food. I Initiating therapy with a low dose of the drug. IM Taking aspirin 30 minutes priorto Niacin. a Tonly b —_TandITonly © MandIlfonly a all 867. Theactive ingredient of Humibid LA is a Dextromethorphan b. Codeine cc. Guaifenesin 4. Benzonatate 868. Whenasubject changes the posture from the lying down position to the upright, there isa reflex hypotension called: Systematic hypotension b. _Asymptomatichypotension ¢. Postural hypotension 4 Aortic hypotension 106 Reference Guide for Foreign Pharmacy Licensing Exam Questions and Answers- 2nd Revised Edition 869. Which of the following drugs is NOT an ‘enzyme inhibitor? a Cimetidine b. Fluoxetine cc. Carbamazepine d. Ciprofloxacin 870. The serum concentration of which of the following neurotransmitter(s) is/are increased in pheochromocytoma ? I Epinephrine I Norepinephrine I Tyramine a Tonly b. — TandIonly c, MandIltonly a All 871. Which of the following is a reversible, nonselective MAO inhibitor ? a. Colesevelam b. Linezolid cc. Fexofenadine 4. Lopinavir/Ritonavir 872. Which ofthe following drug(s) should be avoided with Digoxin? 1 Brythromycin I Quinidine Ill. Colestipol a. Tonly b. Land Ionly c. Hand Ill only dll 873. CalciumAcetate is indicated for treat- mentof www.pharmacyexam.com Krisman a. Hypocalcemia b. —_Hypophosphatemia cc. Hyperealoemia 4. Hyperphosphetamia 874. The principal function of bile salt is to: metabolite carbohydrates emulsify fats inhibit gastric acid secretion and motility provide alkaline pH peoe 875. Thenormal therapeutic range of activated partial thromboplastin time is 12 to 14 seconds a, b. — 20te30 seconds c. 351045 seconds 4. 1 to5 seconds 876. Vancomycin Enemais indicated for reat- ment ofinfections caused by: Chadifficile C. tetanus P.aeruginosa E.Coli eege 877. The normal serum plasma concentration of Potassium is a. 101020 meg/L. b. —-3.5toSmeq/L c. 3510.40 meq/L d > 120meq/L 878. Which of the following isa short-acting barbiturate? Phenobarbital Amobarbital Butabarbital Pentobarbital ees 107 Reference Guide for Foreign Pharmacy Licensing Exam Questions and Answers- 2nd Revised Edit 879. Ritonaviroral solution should be avoided. with a Metronidazole b —__Diphenhydramine © Amitriptyline Alprazolam 880. Clorazepate SD is indicated for treat- mentof a. Hypertension b. Anxiety Insomnia a BPH 881. Administration of which of he following drugs requires caution when using with Fenofi- brate? a Erythromycin b Warfarin © Risperidone d—_Levofloxacin 882. Which of the following drugs should be carefully prescribed with Omnicef (Cefdinir) ? 1 Aluminumhydroxice Ferrous fumarate I Probenecid a Tonly b —_TandITonly ¢ Mand Ifonly ds All 883. Zemplar(Paricalcitol) is indicated for the treatmentof a Hypertension b _Hyperparathyroidism c Diabetes 4 Glaucoma www.pharmacyexam.com Krisman 884. How many grams of Dextrose are re- quired to prepare 5% of 500cc solution ? a -25gm b.-25gm c 55gm a Sm 885. Which of the following is NOT a neu- rotransmitter? a. Epinephrine b. Histamine cc. Dopamine 4d. MAO 886. How much Atropine is required to dis- pense I quart of 1 in 100 solution ? aege 9.6mg 1.48 gm 23mg 9600 mg 887. 20ccof 10% KCI solution, 50 cc of 25% NaHCO3 solution and 30 cc of 20% CaC2 are mixed with 5% 1000cc dextrose solution. The infusion should be administered over 8 hours. What is flow rate in drops/min ? (LV. set delivers eee 0 drops/ec) 13.19 14.05 18.01 22.91 888. Azopt (Brinzolamide) is indicated for the treatment of a Hypertension b Edema © Glaucoma 4 Gout 108 Reference Guide for Foreign Pharmacy. 889. To prepare 0.25% of 1000 ce Dakin so- ution, how many ce of 5% sodium hypochlorite are required ? a 100cc b. 50ee © 60ee d. 40ce 890. ‘Tazorac (Tazarotene) topical cream is in- dicated forthe treatment of 1 Psoriasis I Acne II Poison ivy a Tonly b Land Ionly c Iand I only d All 891. The extended release action of Ditropan ‘X1(Oxybutynin ER) is attributed to its GITS system ‘Wax matrix form Granules formulation Inner coat with outer coat ppoe 892. The initial concentration of drug decom- position according to first order kinetic is 10? units! ml, When the concentration of drug falls below 10 units/cc, it should be removed from market. What would be the expiration date for this prod- uct? (K = 2.303 x 10" hour! ] a 20x 10hours b. 40x 10" hours c. $x 10*hours 4. 40hours 893. Themajoradverse effect of Doxil (Doxo- rubicin)is www.pharmacyexam.com ensing Exam Questions and Answers- 2nd Revised Edition Krisman 1 Cardiac failure I _Liverfailure MI. Renal failure a Tonly b —_TandIonly © Mandl only dll 894, 1£5000mg of triamcinolone powder are mixed with 1 Ib of 1% triamcinolone cream, what is the percentage of triamcinolone in the final mix- ture? a, 1.25 % wiw b. 3.56% wiw ce 21% whw da 4.2% wiw 895. Which ofthe following drugsis indicated for the treatment of Cisplatin toxicity ? Acetyleysteine Antiplatin ‘Ammifostine Naltrexon 896. Whatis the osmolarity of 500 cc of 25% of sodium bicarbonate solution ? (mw=84 gmvmole) a 5952.38 b. 2763.18 ©. 7032.11 4. 1809.09 897. Clotrimazole Troches are indicated forthe treatment of a. Onychomycosis, b. Oral candidiasis, ©. Athlete’s foot 4. Ringworm infection of groin 109 Reference Guide for Foreign Pharmacy Licensing Exam Krisman ‘Questions and Answers- 2nd Revised Edition 898. How many calories are provided by 500 cc of D,,W solution ? a 145 calories b. 510 calories ¢. 325calories d.— 252ealories 899. The disorder of the eyein which the per- son can see near objects perfectly but finds it . GITS ce. ORDS 4. External coat with an internal core of drug 915. The active ingredient(s) of Tenoretic is/ are 1 Atenolol TM —Chlorthalidone Tl Hydrochlorothiazide a Tonly b. Land I only c. Hand Ilf only a all i Reference Guide for Foreign Pharmacy Licensing Exam Questions and Answers- 2nd Revised Edition 916. Zestopril (Lisinopril + HCTZ) should be avoided in patients with Sulfonamide allergy NSAID allergy Opioid allergy TCA allergy pese 917. Ifa prescription for TPN calls for pro- viding 55% of 1500 nonprotein calories as a dex- trose, how many cc of D,,W are required ? a 451.520 b. — 323,32ec c. 808.82 cc d 616.15 cc 918. Whatis the volume of dextrose solution required to prepare a peripheral parenteral nutri- tion that provides 1100 nonprotein calories and 30 gm of amino acids ? The formulation should provide 45% of nonprotein calories as fat and have a maximum dextrose concentration of 125%. a 576.15cc b. 823.18 ec ce. 1423.52ce d. 390.63 ce 919. Zoladex (Goserelin) is indicated for the treatment of @. Hypotension b. Prostate carcinoma ¢ Meloid leukemia 4. Hairy cell carcinoma 920. Zomig (Zolmitriptan) is a/an a. 5-HT receptor agonist b. Dopamine recepicr antagonist ¢. _5-HT3 receptor antagonist 4. Angiotensin Il receptor antagonist www.pharmacyexam.com Krisman 921. Zomig (Zolmitriptan) should not be pre- scribed to patients suffering from the following, conditionsEXCEPT a, Suffering from hypertension or ischemic heart disease. b. Taking MAO inhibitors. c. Taking Dihydroergotamine. 4. Suffering from L pneumonia. 922. WhenAtamet (Levodopa+ Carbidopa) is to be given to patients who are being treated with Levodopa, Levodopa must be discontinued atleast a, 4hours before therapy with Atamet is initiated. b. _ Shours before therapy with Atamet initiated. ¢. 12 hours before therapy with Atamet is initiated. d. 24 hours before therapy with Atamet is initiated. 923. Permax (Pergoloid) is indicated for the treatment of a, Hypertension b. Diabetes cc. Gout d. Parkinson 924. Zanaflex (Tizanidine)isa centrally acting beta-I receptor agonist beta-2 receptor agonist alpha-1 receptor agonist alpha-2 receptor agonist eege 925. Allegra (Fexofenadine) is a(n) a. Histamine H-1 receptorantagonist b. —Alpha-1 receptor agonist 112 Reference Guide for Foreign Pharmacy Licensing Exam ‘Questions and Answers- 2nd Revised Edition €. Dopamine D-1 receptoragonist 4. Histamine H-2 receptor antagonist 926. The nasal decongestant in Allegra-D (Fexofenadine) is a Epinephrine b. Phenylephrine ©. Pseudoephedrine d. Oxymetazoline 927. Which of the following substances can the brain utilize for energy production? a Fat b. Glucose c. Fatty acids d. Amino acids 928. Anzemet (Dolasetron) is indicated for the treatment of a. Hypertension bd. BPH c. Prevention of nausea and vomiting associated with emetogenic cancer therapy. d. Diabetes 929. Anzemet Dolasetron) therapy shouldbe monitored by a BCG b. CAT scan c. COMB'sTest d.— Antinuclear Antibody Test 930. The active ingredient of Arava is Etoposide Leflunomide a : Voltaren, Rose www:pharmacyexam.com Krisman 931. The most frequently reported adverse ef- fect with Carafate (Sucralfate) is a. Diatthea b. Vomiting c. Constipation 4. Bleeding 932. Clomid (Clomiphene) is indicated for the treatment of Ovulatory dysfunction Hypertension Atherosclerosis Rheumatoid arthritis aege 933. DDAVP(Desmopressine) isa synthetic analogue of a. thyroid hormone b. pituitary hormone ¢. _gonadotropic hormone 4d. parathyroid hormone 934, Which ofthe following are correct DEA numbers for Dr. Sheghi, Rama ? a. BS4312567 b. —AR3252181 c. BR4530895 d—-AS2332764 935. The duration of action of Lantus (Insulin glargine)is a. 4hours b.— Thour c. 24hours 4. 30minutes 936. How many sperm is/are produced from primary spermatocyte? a One 3

You might also like